Pharmacology Final

Ace your homework & exams now with Quizwiz!

What action is needed for a blood magnesium level of 4.8 mg/dL?

CALCIUM GLUCONATE IS INDICATED AS THE IV ANTIDOTE FOR EXCESS MAGNESIUM FOR THIS PATIENT.

Match the antibiotic with the class that it belongs to: IMIPENEM-CILASTATIN

CARBAPENEMS

What source is the most accurate for specific recommendations for childhood immunizations?

CDC

Match the following medications with the most accurate drug classification: TRAMADOL (ULTRAM)

CENTRALLY ACTING DRUG

Match the antibiotic with the class that it belongs to: CEPHALEXIN

CEPHALOSPORIN

Match the Antibiotic with the class that it belongs to: CEFTRIAXONE

CEPHALOSPORINE

Match the following cancer and cancer treatment terms with the correct definition: PALLIATION

CHEMOTHERAPY DRUGS ADMINISTERED TO EASE SEVERITY OF PAIN AND OTHER TUMOR SYMPTOMS

Match the medications listed below with the complication of chemotherapy for which it is prescribed: LORAZEPAM

CHEMOTHERAPY INDUCED NAUSEA AND VOMITING

Match the medications listed below with the complication of chemotherapy for which it is prescribed: ONDANSETRON

CHEMOTHERAPY INDUCED NAUSEA AND VOMITING

Match the following COPD conditions with the best definitions: ASTHMA

CHRONIC PULMONARY DISEASE WITH INFLAMMATORY AND BRONCHODILATION

Match the Category with its fluid replacement agent: DEXTRAN 40

COLLOID

Match the Category with its fluid replacement agent: PLASMA PROTEIN FRACTION

COLLOID

The blood bank is going to separate out component parts of blood. Choose which part of the blood matches which description: FRESH FROZEN PLASMA

CONTAIN CLOTTING CLOTTING FACTORS (EG. FACTOR VIII)

Match the following medications with the most accurate drug classification: CELECOXIB (CELEBREX

COX 2 INHIBITORS

Match the following medications with the most accurate drug classification: IBUPROFEN

COX-1 AND COX-2 INHIBITOR

Match the following medications with the most accurate drug classification: INDOMETHACIN (INDOCIN)

COX-1 AND COX-2 INHIBITORS

H2 Antagonists like Ranitidine may elevate all the following values except?

CRP

Match the Category with its fluid replacement agent: D5W

CRYSTALLOID

Many patients with cancer experience a general wasting of muscle and other tissues referred to as

Cachexia

What are the most common types of calcium supplements that can be purchased OTC?

Calcium citrate Calcium carbonate

Food-drug interactions: (choose all that apply)

Can prevent a medicine from working as it should Can increase or decrease side effects a patient is already experiencing May cause new side effects not experienced by a patient before Are mild enough that patients can take their medications as prescribed and resume their normal diets Can change how a patient's body metabolizes food (ALL OF THE ABOVE)

Inotropic drugs are primarily used for what type of shock?

Cardiogenic

Why are bone marrow suppression, diarrhea and alopecia so common when clients are given anticancer agents?

Cells lining the GI tract, RBCs, WBCs and hair follicle cells are replaced at a very fast rate

Of the following which is largest class of antibiotics?

Cephalosporins

When doing a brown bag evaluation of Mr. Allen's medications the nurse notices the patient takes digoxin and Ginseng. It will become important to:

Check the patient's heart rate for potential digoxin toxicity

In taking a new patient's history, the nurse notices that he has been taking omeprazole consistently for over the past 6 months for treatment of epigastric pain. The nurse will recommend:

Checking with his provider about his continued epigastric discomfort

A patient preparing for a colonoscopy is prescribed a clear liquid diet. What foods should she take? (Choose all that apply):

Chicken broth Lemon jello apple juice white grape juice (All but red jello)

Some medications when prescribed for children must be given more frequently than for adults. What explanation is correct?

Children have a faster basic metabolic rate than adults.

Most drugs used to treat Alzheimer's disease affect

Cholinergic pathways, enzymes, and receptors

A patient with asthma who does not tolerate the beta 2 agonists is prescribed a drug to block the parasympathetic response rather than stimulating the sympathetic nervous system. This class of drugs is?

Inhaled anticholinergics

Most anti-cancer agents act by

Inhibiting Cell proliferation

Which statement by a family member of a patient diagnosed with Alzheimer's disease demonstrates the medication education was effective? "The medication my parent is receiving:

Inhibits breakdown of an important neurotransmitter and may slow disease and progression

Match the Antibiotic with the class that it belongs to: AMOXICILLIN

PENICILLIN

Match the antibiotic with the class that it belongs to: PIPERACILLIN

PENICILLIN

What is the mechanism of action of pantoprazole sodium (Protonix) in the stomach?

Inhibits the proton pump in the stomach lining from creating more acid

Atomoxetine selectively...................the presynaptic reuptake of norepinephrine resulting in a(n)......................attention span

Inhibits, improved

Match drug with the location of action it targets: LATE DISTAL TUBULE AND COLLECTING DUCTS

POTASSIUM SPRING DIURETICS

Choose the actions of the following medications: ONDANSETRON

PREVENTS NAUSEA AND VOMITING ASSOCIATED WITH CHEMOTHERAPY, RADIATION, AND SURGERY

Mach the drug to the mechanism of action: SITAGLIPTIN

PREVENTS THE BREAKDOWN OF INCRETINS

Match the following cancer and cancer treatment terms with the correct definition: STAGING

PROCESS OF DETERMINING WHERE THE CANCER IS LOCATED

Match the stomach acid reducing medication with its classification: OMEPRAZOLE

PROTON PUMP INHIBITOR

Match the medication with its classification: POLYETHYLENE GLYCOL

PULLS FLUID INTO THE LUMEN OF THE BOWEL TO CREATE A MORE WATERY STOOL

Metformin is used off-label to treat which of the following common problems?

Polycystic ovary disease

A 3-year-old child comes to the clinic for an overdose of pseudophedrine. For what signs of toxicity should the nurse be watching? Select all that apply

Insomnia TACHYCARDIA Seizures

A patient being treated for DKA received an Insulin infusion, Sodium Bicarbonate, and D5W. Why did this patient receive D5W?

Insulin will facilitate glucose metabolism so serum glucose may drop too quickly

You are taking care of a burn patient with 60% of his body burned from a car accident. The nurse knows that in order to keep the patient hemodynamically stable the priority nursing care is:

Intensive fluid resuscitation with LR

The............form of ziprasidone is frequently used for control of acutely agitated patients.

Intramuscular (IM)

The most common anticholinergic prescribed for asthma is

Ipratropium

A pregnant patient is encouraged to take a pre-natal vitamin. She is complaining of constipation> which vitamin is the culprit?

Iron

Which assessment finding is a priority concern when a client has taken pseudoephedrine (Sudafed)?

Irregular heart rate

What is normal saline?

Isotonic solution of 0.9% NaCl

A client has been prescribed cromolyn (Intal) a Mast cell stabilizer and asks you how this medication helps. What is the best response by the nurse?

It acts by inhibiting the release of histamine from mast cells.

Why is Metformin contraindicated for 2 days prior to and 2 days after receiving IV radiographic contrast?

It can result in lactic acidosis

The physician orders pentoxifylline (Trental) for the patient with peripheral vascular disease. The nurse has completed medication education and determines that learning has occurred when the patient makes which statement(s)? (Select all that Apply)

It decreases my platelets so my blood is less likely to clot It makes my red blood cells (RBCs) flexible so they can go into the little blood vessels It decreases the inflexibility of my blood

The patient receives enoxaparin (Lovenox) postoperatively. The nurse teaches the patient about this medication and evaluates that learning has occurred when he makes which statement?

It increases the time it takes for me to form a clot

What is a leukotriene?

It is an inflammatory mediator produced in leukocytes and other immune cells

Which of these statements about the anticholinergic drug, Ipratropium (Atrovent) is correct?

It is approved for the symptomatic relief of runny nose, but does not have decongestant action

Which of the following statement is correct regarding Insulin?

It is required for the cellular uptake of glucose in all cells except those in the brain

Why does Dextran 40 stay in the veins after being given?

Its molecules are too big

What is an example of a sensory cue (auras) that a patient may experience if a migraine attack is imminent?

Jagged lines or flashing lights in the field of vision

Considerations for teaching patients about self-administering medications at home may need to include instructing the patient to take the medication: (Choose all that apply)

Just after eating Upon first waking up or going to bed with extra water but not consuming caffein (everything but with alcohol)

Teaching for a parent prior to them assisting a child with topical permethrin application should include (choose all that apply.)

Keep this out of your child's eyes, as it will cause irritation You can purchase this drug OTC You may need to treat your child within nine days after the eggs hatch.

Fluticasone (Flonase) is contraindicated in patients with a history of which of the following?

Known bacterial infection

Match the following: ISOTONIC

LACTATED RINGERS

The blood bank is going to separate out component parts of blood. Choose which part of the blood matches which description: ALBUMIN

LARGE (PROTEIN) MOLECULES FOUND IN BLOOD, OFTEN GIVEN TO CIRRHOSIS PATIENTS WHO ARE AT RISK FOR COMPROMISED IMMUNE

The blood bank is going to separate out component parts of blood. Choose which part of the blood matches which description: WBC'S

LEUKOCYTES

Match the drug with its classification: MONTELUKAST

LEUKOTRIENE MODIFIER

Match the drug with its classification: ZAFIRLUKAST

LEUKOTRIENE MODIFIER

Match the interaction of sodium bicarbonate with the following medications: MILK OR CALCIUM SUPPLEMENTS

LIKELY TO CAUSE HYPERCALCEMIA AND KIDNEY FAILURE

Match with drug class: BUMETANIDE

LOOP DIURETIC

Match with drug class: FUROSEMID

LOOP DIURETIC

Match drug with the location of action it targets: ASCENDING LOOD OF HENLE

LOOP DIURETICS

When taking ACE inhibitors, patients should be instructed to avoid: (select all that apply)

Leafy greens like kale bananas Salt substitutes like LoSalt Oranges

A patient is on warfarin for atrial fibrillation what antibiotic should be avoided?

Levofloxacin

Pancrelipase is made up of the following enzymes: (choose all that apply)

Lipase Protease Amylase

Mr. Robbins is placed on quetiapine, which has the potential side effects of elevating cholesterol. Which laboratory test should you monitor related to this side effect?

Lipid profile

A patient has been receiving NS to correct dehydration due to diarrhea she's had for the past two days. When the patient complains of tightness in the chest and slight swelling in the fingers, what can the nurse do next and why?

Listen to the lungs. Edema may be developing in the lungs due to the fluid administration

Venlafaxine is extensively metabolized on first pass through the ..................

Liver

Which organ manufactures many of the clotting factors required for blood clotting?

Liver

One advantage of Donepezil is

Long half life

Which of the following is not a side effect of taking medications for fecal regulation such as docusate, senna, or polyethylene glycol?

Lowering LDL cholesterol levels to adverse levels

ACE inhibitors reduce symptoms of heart failure by:

Lowering blood pressure

Match the Antibiotic with the class that it belongs to: ERYTHROMYCIN

MACROLIDES

Match the drug with its classification: CROMOLYN

MAST CELL STABILIZER

Match the interaction of sodium bicarbonate with the following medications: PSEUDOEPHEDRINE AND EPHEDRINE

MAY DECREASE ELIMINATION AND THEREBY CAUSE MORE CNS SYMPTOMS OF DROWSINESS

Match the interaction of sodium bicarbonate with the following medications: KETOCONAZOLE

MAY DECREASE THE EFFECTIVENESS OF THIS ANTIFUNGAL

Patients taking colchicine should have their blood levels tested for which of the following abnormal values?

MCV (size of RBCs) (colchicine can cause Vit. B-12 anemia. RBCs would appear macrocytic and malformed).

Match the drug with its classification: AMINOPHYLLINE

METHYLXANTHIN

The nurse is instructed to give a patient with mucositis "magical mouthwash". The nurse understands that this magical cocktail includes which of the following to decrease the pain. Select all that apply.

Maalox Viscous lidocaine Liquid diphenhydramine

What teaching should you provide to a patient on long term isoniazid?

May need periodic lab work to check for hepatotoxicity

Which patient group can we expect will have problems maintaining their levels of vitamin D?

Patients on dialysis

Match the antibiotic with the class that it belongs to: AMPICILLIN

Penicillin

Which of the following antibiotics is most commonly used for respiratory infections?

Penicillin

For which patients are SGLT2 inhibitors prescribed?

People with type 2 diabetes that have hyperglycemia despite use of other hypoglycemic medications.

What kinds of patients may benefit from taking alendronate? choose all that apply:

Post-menopausal women Aging individuals Individuals taking steroids, like prednisone for long periods of time

Which side effect doesn't diminish after extended opioid use?

Constripation

What is the mechanism of action for metoclopramide (Reglan), which is commonly prescribed for nausea and vomiting?

Contracts the muscles in the upper intestine, resulting in faster emptying of the stomach into the small bowel.

A patient with mucositis form chemotherapy asks the nurse what types of food may help with the pain from the ulcerations in his mouth and esophagus. The nurse instructs the patient to eat:

Creamy peanut butter, puddings, and custards

Which of the following is used to guide dosing of drugs with potentially decreased renal elimination in elderly patients?

Creatinine clearance

Match the Category with its fluid replacement agent: LR's

Crystalloid

Match the Category with its fluid replacement agent: NORMAL SALINE

Crystalloid

Which fluid class do we give most often in the hospital for those with dehydration of any cause?

Crystalloids (like normal saline, which is given routinely for dehydration)

Mach the drug to the mechanism of action: METFORMIN

DECREASE THE HEPATIC PRODUCTION OF GLUCOSE PRIMARILY AND REDUCES INSULIN RESISTANCE SECONDARILY

Mach the drug to the mechanism of action: INSULIN

DECREASES BLOOD GLUCOSE LEVELS

Match the medication with its classification: DOCUSATE SODIUM

DECREASES THE SURFACE TENSION BETWEEN OIL AND WATER IN FECES, ALLOWING FLUID O PENETRATE STOOL, SO STOOL IS SOFTER

Mach the drug to the mechanism of action: ACARBOSE

DELAYS DIGESTION OF GLUCOSE IN THE SMALL INTESTINE

Choose the actions of the following medications: SIMETHICONE

DISPERSES POCKETS OF GAS IN THE GI TRACT

Taking a diuretic like spironolactone can affect levels of:

Potassium

A patient is being started on Amioderone what is the most concerning lab value

Potassium 3.1

What is the classification of Amioderone?

Potassium Channel Blocker

What immunizations are recommended to be given at 4-6 years prior to entering school?

DTaP, IPV, MMR, Varicella

What health teaching would be most appropriate for a patient started on nifedipine?

Daily weights

Because diazepam and flurazepam have half-lives up to 96 hours in elderly patients. When may signs of toxicity appear?

Days or week after therapy is started.

When giving a patient KCL through her nasogastric tube the nurse dilutes the medication prior to administration. Why does she do this?

Decrease GI upset

Patients who begin taking colchicine should be taught to avoid eating grapefruit and drinking grapefruit juice as it may:

Decrease hepatic metabolism of colchicine

Simethicone may be prescribed in order to:

Decrease the amount of gas associated with some GI disorders

What is the priority electrolyte concern for a patient receiving treatment for DKA?

Potassium levels dropping

Mike a 62 yr old male started taking Propranolol for dysrhythmias. What side effect of propranolol is a major risk factor for lack of compliance?

Decreased libido

What physiologic change associated with normal agin may affect an older adult's response to drugs?

Decreased responsiveness of the carotid baroreceptors

Singer Michael Jackson allegedly abused alprazolam and propofol, an anesthetic. Which interference applies?

Potentially lethal sedation and CNS depression would be expected with this drug combination

Which of the following medications is a synthetic analog of a natural hormone released by the beta cells of the pancreas at the same time as insulin.

Pramlintide (Symlin)

For which of the following antilipemics are grapefruit and grapefruit juice contraindicated?

Pravastatin

Thiazolidinediones (TZDs) such as rosiglitazone and pioglitazone lower blood glucose by which of the following mechanisms?

Decreases insulin resistance and inhibits hepatic gluconeogenesis

Which of the following oral hypoglycemics is preferred for managing Type II Diabetes because it is safe, effective and it does not cause hypoglycemia

Metformin (Glucophage)

A nurse will prepare teaching materials regarding which drug for the parents of a child diagnosed with ADHD?

Methylphenidate

A patient is experiencing gastroparesis, which medication would be prescribed for this patient?

Metoclopramide (Reglan)

A patient with ventricular dyrhythmias is receiving lidocaine. What is the most appropriate intervention for this therapy?

Monitor for paresthesisas, drowsiness or confusion

A nurse administering IV calcium should consider: Choose all that apply.

Monitor serum calcium levels to avoid hypercalcemia Administer slowly to avoid hypotension dysrhythmias and cardiac arrest. Monitor IV site closely for signs of infiltration

The nurse getting ready to start an infusion of milrionone knows once she starts the medication she will need to

Monitor the ECG continuously

Why is most insulin today human insulin? Check all that apply

More effective Fewer allergies Lower incidence of resistance (everything but the option of giving orally. Cant be given orally)

Naloxone (Narcan) inhibits which of the following pain receptors?

Mu and kappa

The risk of experiencing serotonin syndrome when selective serotonin reuptake inhibitors (SSRIs) are given with monoamine oxidase inhibitors (MAOIs) such as phenelzine. Serotonin syndrome is best characterized in which of the following?

Muscle rigidity and high fever

The nurse knows that the most common reason for delaying chemotherapy is due to which common side effect of chemotherapy?

Myelosuppression

Match the medications listed below with the complication of chemotherapy for which it is prescribed: FLIGRASTIM

NEUTROPENIA

Hypovolemic shock can be triggered by which of the following conditions: (choose all that apply)

NG suctioning Vomiting Diarrhea Intensive diuretic therapy

Match the following drug with its indication for use: ICE PACK

NON-PHARMACOLOGIC PAIN MANAGEMENT

Match the following medications with the proper drug classification: KETORALAC

NSAID

Patients with diarrhea should be aware that taking Diphenoxylate with atropine (Lomotil) may lead to which of the following? Check all that apply

Narrow angle glaucoma CNS depression Continued alcohol consumption (everything but diarrhea. Lomotil treats diarrhea)

Pseudoephedrine is primarily used to treat which of the following conditions?

Nasal congestion

Meglitinides include which of the following medications? Check all that apply

Nateglinide Repaglinide

Cyclobenzaprine.................muscle activity primarily in the...................

Depresses, brainstem

A nurse administering promethazine for a patient's nausea understands the risks of IV administration include: (choose all that apply)

Necrosis to the vein May lead to gangrene and amputation of affected limb Caustic impact to the intima of the vein Spasms of blood vessels Swelling to vein and tissues

A patient is taking an IV aminoglycoside, gentamicin, to treat a severe infection. His urine is dark, his urinary output has diminished, he is having a hard time hearing you, and he is scratching his arm. Which side effects can be attributed to taking gentamicin?

Nephrotoxicity Ototoxicity

A side effect that can be life threatening and is common to many antibiotics is:

Diarrhea

Anticholinergic drugs provide symptomatic relief for some patients with IBS. Which of the following is an anticholingergic drug used for IBS?

Dicyclomine (Bentyl)

Taking calcitriol is not completely risk free. Which of the following can occur when taking calcitriol with a particular drug? Select all that apply.

Digoxin: dysrhythmias Magnesium: hypermagnesemia Thiazide diuretics: hypercalcemia

An administration consideration the nurse must recognize for promethazine hydrochloride is:

Dilute IV doses in at least 10 mL of normal saline and give slow IV push over 2 minutes or more.

Proper administration of IV promethazine includes:

Dilute in the appropriate amount and utilize a slow push method, over at least 2 minutes or more.

Powdered potassium chloride supplements need to be given:

Diluted in water or juice

When a patient is prescribed risperidone for schozoaffective disorder, the nurse must alert the patient to the potential for neuroleptic malignant syndrome. Which of the following would be accurate information to give the patient?

Neuroleptic malignant syndrome is potentially fatal. It usually occurs during the first 2 weeks after starting an antipsychotic drug or increasing the dose and is characterized by high fever, muscle stiffness, paranoia, sweating, and large changes in blood pressure

Match the stomach medication with its action: ALUMINUM HYDROXIDE

Neutralizes stomach acid and raises the PH of the stomach

Which supplement is beneficial for lowering overall cholesterol?

Niacin

Which over-the-counter drugs has the least risk of side effects?

No over-the-counter medications are safe to take.

Patients being treated for allergic rhinitis with nasal corticosteroids should be monitored for:

Non-intact nasal mucosa

Following an acute myocardial infarction, your patient is receiving alteplase (Activase), a thrombolytic drug. Which of the following adverse effects could be attributed to this drug administration?

Nosebleeds

A patient with open angle glaucoma who has been taking latanoprost drops in her eyes complains of severe pain in one eye, severe headache, and blurred vision in both eyes. What should the nurse do in response to this?

Notify the healthcare provider immediately.

Match the following medications with the most accurate drug classification: HYDROMORPHONE

OPIOID AGONIST

Match the following medications with the proper drug classification: METHADONE

OPIOID AGONIST

Match the following medications with the proper drug classification: NALAXONE

OPIOID ANTAGONIST

Match the medication with its classification: DIPHENOXYLATE WITH ATROPINE

OPIOID ANTI-DIARRHEAL

Match the following Pharmacotherapy for allergic rhinitis: PSEUDOPHEDRINE

ORAL DECONGESTANT

Match drug with the location of action it targets: PROXIMAL TUBULE AND LOOP OF HENLE

OSMOTIC DIURETICS

Match the medication with its classification: POLYETHYLENE GLYCOL

OSMOTIC LAXATIVE

What instructions would the nurse give a patient with a new prescription for timolol eye drops? check all that apply

Occlude the inner canthus of the eye for a minute after the drop is given Avoid touching the tip of the bottle to the eyelid or eye Wash your hand before giving yourself these drops

A patient was started on fluoxetine for depression 3 days ago. She ow complains that it "doesn't help" and refuse to take it. Understanding psychoparhmacology, the nurse's best first action is to:

Discuss with the patient that these types of medcations often take several weeks before seeing the resutls

Valporic acid can cause a ...............Drug-drug interaction with warfarin

Displacement

Choose all that apply. Which of the following interventions are considered non-pharmacological pain relievers?

Distraction, like music Positioning of patient Heat or cold packs

A patient diagnosed with mania begins lithium therapy on October 1. What is the earliest date the nurse can expect to see clinical improvement in the patient's condition?

October 8

When educating a patient who is starting to take dextromethorphan (Robitussin) what would be the main teaching points the nurse should discuss regarding adverse effect associated with taking the "suggested dose" of this medication?

Dizziness, drowsiness, GI upset

What is important to know about azithromycin?

Do not combine with alcohol.

As the patient leaves your clinic with his new prescription of inhaled ipratropium he tells you that he is going on vacation to St. George and asks you how to store the medication. The best intruction by you is

Do not leave it in your car while traveling

Teaching for patients taking tretinoin include: (choose all that apply)

Don't use other OTC acnes medications or skin products that will dry your skin If you are allergic to fish, you shouldn't take this Avoid direct sunlight and use sunscreen, as you are more likely to have skin damage when using this

A significant number of patients taking metoclopramide (Reglan) may have which of the following adverse CNS effects?

Drowsiness, fatigue, and insomnia

Indications for the use of an antitussive, such as dextromethorphan include:

Dry, hacking cough

Choose the definition of an isotonic solution

Of the same concentration as the blood

Match the common adverse effects with the correct cause: MUCOSITIS

EPITHELIAL LINING OF DIGESTIVE TRACT BECOMES INFLAMED

The blood bank is going to separate out component parts of blood. Choose which part of the blood matches which description: RED BLOOD CELLS

ERYTHROCYTES

Match the following COPD conditions with the best definitions: CHRONIC BRONCHITIS

EXCESS MUCUS IS PRODUCED IN THE LOWER RESPIRATORY TRACT

Match the following Pharmacotherapy for coughs: GUAFENESIN

EXPECTORANT

Because the epithelial lining of the GI tract is very sensitive to chemotherapy, the nurse anticipates teaching the patient

Eat a popsicle during chemotherapy infusions

What is an important teaching point for patients receiving warfarin therapy?

Eat the same amount of food each week that are rich in Vitamin K (asparagus, broccoli, cabbage, cauliflower, kale)

What is an important teaching point for patients receiving warfarin therapy?

Eat the same amount of food each week that are rich in vitamin K (asparagus, broccoli, cabbage, cauliflower, kale)

Why do serum creatinine levels often remain within normal limits as we age despite a decrease in GFR?

Elderly generally have less muscle mass and are generally less physically active then younger adults. (because of this they produce less creatinine)

SGLT-2 inhibitors (Gliflozins) include which of the following medications? Select all that apply.

Empagliflozin Canagliflozin Dapagliflozin (all the flozins all but sitagliptin)

What is the most important teaching point for the patient with COPD

Encourage smoking cessation

Which of the following is considered a low molecular weight heparin?

Enoxaparin (Lovenox)

When teaching a patient about a new prescription for Lisinopril the nurse knows to include which statement in his/her teaching?

Excessive intake of potassium should be avoided

Which of the following is a common side effect of both first- and second-generation antipsychotic drugs?(select all that apply)

Excessive sedation Neuroleptic malignant syndrome Extrapyramidal side effects such as pseudo-parkinsonism or acute dystonia

A patient with a sodium level of 151 mEq/L. What would be the expected actions from the nurse? (choose all that apply)

Expect an order for the patient's diet change to a low-sodium diet Notify the doctor of this value

Match the Antibiotic with the class that it belongs to: LEVOFLOXACIN

FLOUROQUINOLONES

Match the antibiotic with the class that it belongs to: CIPROFLOXACIN

FLOUROQUINOLONES

A patient who loses 15% of his blood supply due to hemorrhage will most likely receive packed red blood cells to help improve oxygen carrying capacity of his system

False

Acetaminophen is the only analgesic which may be taken with alcohol (T/F)

False

Antineoplastic medications are more likely to cause permanent sterility in women than in men.

False

Aspirin (ASA) and ibuprofen only inhibit COX-1 celecoxib (Celebrex) only inhibits COX-2 (T/F)

False

Bethanchol is used to decrease smooth-muscle tone and muscular contractions in the GI tract following general anesthesia. (T/F)

False

Chemotherapy is preferred over surgery or radiation because a combination of drugs will kill 100% of the tumor cells

False

Cromolyn is a mast cell stabilizer used for treatment of asthma only (True/False)

False

Dysphoria from opioid treatment can include side effects of restlessness, depression and hallucinations

False

For the first four hours after initiating opioid therapy, the nurse will assess the RR every 1-2 hours (T/F)

False

Ibuprofen should be considered a safe, over-the-counter drug with very few side effects

False

Levodopa and Carbidoba can be given as individual medication and still have the same therapeutic effect as Sinemet (T/F)

False

Methylphenidate is not effective for narcoleptic patients

False

Most anticancer drugs act by killing cells that are dividing, thus non cancer cells (normal cells) are not destroyed during cancer treatment

False

Phenelzine does not need to be discontinued several days before elective surgery that requires anesthesia

False

Taking food with buspirone alters absorption

False

The actions of acetaminophen include its analgesic, antipyretic, and anti-inflammatory properties (T/F)

False

Trazadone is more effective as an antidepressant rather than a sleep aid (T/F)

False

True or False. Albuterol has anti-inflammatory properties.

False

True or False: Pharmacotherapy works well for patients with irritable bowel syndrome

False

True or False: Roflumilast is a new drug approved in 2011 to cure COPD

False

When teaching a client about opioid therapy, the nurse will encourage the patient to take the drug only when the pain is severe (T/F)

False

When the patient's respiratory rate is 12, administering naloxone would be an appropriate nursing action (T/F)

False

Possible ocular effects of taking hydroxychloroquine include blurred vision, photophobia, diminised ability to read, and blacked-out areas of the visual fields. However, even with high doses or prolonged therapy with visual field changes, these retinal symptoms can be reversed

False (with prolonged usage, retinal changes may be irreversible in some patients. Keep an eye on those eyes...)

Phenylephrine can be administered intranasal, ophthalmic, intramuscular, subcutaneous, and intravenous. With these options IV is the preferred route for this medication (T/F)

False (IV is not preferred due to severe reactions that may occur with IV)

Benztropine is used for relief of Parkinson's like symptoms, extrapyramidal symptoms, and tardive dyskinesia

False (benztropine is not useful for tardive dyskinesia)

T/F Calcitonin is considered a first-line treatment for osteoporosis

False (calcitonin is a 2nd-line Tx first line would be calcium, vi. D, and magnesium supplements and behavioral changes)

Diazepam rapidly absorbs from both the GI tract and IM sites (T/F)

False (diazepam absorbs rapidly from GI but this is not true for IM)

A patient has been prescribed a selective serotonin reuptake inhibitor (SSRI) for depression. The nurse should counsel the patient about which of the following side effects, among others?

Fever, sweating, agitation, rapid heartbeat

Which of the following are colony-stimulating factors used to increased the WBC count. Check all that apply

Filgastrim Sargramostim

Phenobarbital, propranolol and nitrates are metabolized before the drug reaches the systemic circulation and is also affected by aging. This phenomenon is called:

First-pass metabolism

Which of the following are inhaled corticosteroids? (Check all that apply)

Fluticasone Beclamethasone Triamcinilone (all those ending in -one)

Choose the action and purpose of the elements given: DEXTRAN 40

GIVEN AS A COLLOID TO PULL FLUID BACK INTO THE VASCULAR SPACE THEREBY HELP BLOOD PRESSURE INCREASE

The blood bank is going to separate out component parts of blood. Choose which part of the blood matches which description: SODIUM CHLORIDE

GIVEN AS A COMMON CRYSTALLOID TO PATIENTS TO PROMOTE URINARY OUTPUT AND DECREASE THE EFFECTS OF DEHYDRATION

The blood bank is going to separate out component parts of blood. Choose which part of the blood matches which description: POTASSIUM CHLORIDE

GIVEN AS AN ELECTROLYTE REPLACEMET

The blood bank is going to separate out component parts of blood. Choose which part of the blood matches which description: SODIUM BICARBONATE

GIVEN TO HELP CORRECT ACID BASE IMBALANCES THAT CAN OCCUR IN PATIENTS

Match the Antibiotic with the class that it belongs to: VANCOMYCIN

GLYCOPEPTIDES

A patient began trihexyphenidyl for treatment of Parkinson's disease. Which finding demonstrates a positive response to the medication?

Gait is steady with decreased rigidity

Which of the following antibiotics causes ototoxicity?

Gentamycin

For what condition is a patient prescribed latanoprsot

Glaucoma

For what disorder is allopurinol prescribed?

Gout

A patient on warfarin is having two to three nose bleeds a day and is feeling weak after five days of this condition. He comes into the ED to have his warfarin levels checked. What would the nurse expect his lab value for an INR to be?

Greater than 4

Match the stomach acid reducing medication with its classification: RANITIDINE HYDROCHLORIDE

H-2 RECEPTOR ANTAGONIST

Match the common adverse effects with the correct cause: ALOPECIA

HAIR FOLLICLES ARE DAMAGED

Match the common adverse effects with the correct cause: INFERTILITY

HARM CAUSED TO THE GONADS

Methylxanthines such as theophylline and aminophylline are infrequently prescribed because they:

Have a narrow safety margin

Riosiglitazone and pioglitazone contain a black box warning of which of the following?

Heart failure and increased risk for myocardial ischemia

Prior to administering Digoxin to a patient, which measurement is imperative for a nurse to assess?

Heart rate

Methylphenidate is prescribed to an 8-year-old child for the treatment of attention-deficit/ hyperactivity disorder (ADHD). the nurse plans to monitor which of the following during this medication therapy?

Height and weight (stimulants may cause slow growth and reduce appetite)

Which of the following medications may be prescribed for a patient experiencing nausea? (Check all that apply)

Ondansetron (Zofran) Promethazine (Phenergan) Metoclopramide (Reglan)

Donepezil may take ............... before improvement in baseline behavior is observed

One month

Which of the following analgesics are effective at suppressing the cough reflex and slowing the motility of the GI tract?

Opiates

Which of the following lab values is the best indicator of the average blood glucose levels over the past 3 months?

Hemoglobin A1C (HbA1c)

For which of the following is protamine sulfate used as an antagonist?

Heparin

The client receiving heparin therapy asks how the "blood thinner" works. What is the best response by the nurse?

Heparin does not thin the blood but prevents clots from forming as easily in the blood vessels

Clozapine may cause: (Select all that apply)

Orthostatic hypotension Cardiac arrest Syncope

What immunization is typically given at birth?

Hepatitis B

Which of the following immunizations is for Haemophilus influenza type B

Hib

First- pass metabolism is affected by agin and decreased by about 1% per year after the age of 40. Thus, for a given oral dose of nitrates the elderly may have:

Higher circulating drug levels

A patient receives the following medications at 0800. Which medication does the nurse recognize as the reason for the patient feeling chilled, clammy and shaking when he checks on the patient at 1005?

Humalog Regular Insulin, 5 units, SQ (peak time for regular insulin is 2-4 hours after administration)

What medication is a direct acting vasodilator?

Hydralazine

Prazosin can be used for the following conditions:

Hypertension

Which side effect associated with the use of oral sympathomimetic, such as pseudoephedrine, for nasal congestion should the nurse be concerned about?

Hypertension

What is the most common adverse effect of sulonylureas?

Hypoglycemia

Choose the actions of the following medications: METOCLOPRAMIDE

INCREASES THE MOTILITY OF THE UPPER GI TRACT

Match the following Pharmacotherapy for allergic rhinitis: IPRATROPIUM

INTRANASAL ANTICHOLINERGIC

Choose the definition of a crystalloid:

IV fluids that contain electrolytes in concentrations resembling those of extracellular fluid

Choose the definition of a colloid

IV fluids that contain proteins or other large molecules

Your patient on insulin therapy has pale cool, moist skin and is confused. What is the quickest way to reverse these symptoms?

IV glucose in a dextrose solution

A known vesicant is started on a patient with cancer. The nurse will monitor for:

IV site or port for pain, redness and swelling

Drug -Drug interactions for the use of Raloxifene include: (mark all that apply)

Ibuprofen Black cohosh or other estrogen-related drugs ampicillin Warfarin

A patient recently started on linagliptin, an incretin enhancer (DPP-4 inhibitor) as monotherapy. What is the best instruction by the nurse?

Immediately report any itching , rashes, or swelling of the face or tongue

A patient was administered 100mg of trazodone at 10000 instead of at 2200 as ordered. What are the possible outcomes of this administration? (Mark all that apply)

Impaired healing increased risk of falls Decreased nutrition

When treating shock what is the desired outcome of any drug used?

Increase tissue perfusion

Methylphenidate is a CNS stimulant and is the drug of choice for individuals with ADHD. The desired effects of methylphenidate may be noted as

Increased attention span and concentration

Elderly patients are particularly sensitive to anticholinergic drug effects. When a patient takes a drug such as diphenhydramine which has anticholinergic effects what are your concerns?

Increased confusion

Interactions with drugs may occur when a patient is also taking (choose all that apply

Over-the-counter vitamins

A patient taking a statin calls the office to report an increase in muscle pain. What should the nurse communicate next with the patient?

"This could be the sign of a serious side effect, stop taking the medication immediately."

The family of a patient diagnosed with Alzheimer's disease asks the nurse, "How can drugs help our parent?" The reply that provides the most realistic expectations would be:

"drugs can help preserve mental abilities for a time."

Match the following: HYPOTONIC SALINE

0.45% NS

Match the following: ISOtONIC NORMAL SALINE

0.9% NS

How soon before elective surgery should NSAIDs be discontinued?

1 week

At what age is it recommended that an adolescent receive a booster of DTaP vaccine?

11-12 years

At what age is the meningococcal vaccine often given?

18 years

Match the antibiotic with the class that it belongs to: DIPHENHYDRAMINE

1ST GENERATION H1 ANTAGONIST

How many doses of MMR vaccination are recommended for children?

2

How many doses of the varicella vaccination are recommended?

2

Match the following Pharmacotherapy for allergic rhinitis: CETIRIZINE

2ND GENERATION H1 AGONIST

A 23-year-old male who has a history of a cold in the last week comes to the clinic complaining of priapism. What is most likely the cause of his problem?

3 Red Bull energy drinks consumed in the last 24 hours

How long will migraine HA subside after muscles are blocked with onabotulinumotoxin A (botox)?

3 months

Match the following: HYPERTONIC SALINE

3% NS

The recommended course of treatment for intranasal decongestant/sympathomimetic such as oxymetazoline (Afrin) is:

3-5 days

How many doses of DTaP immunization are recommended for young children?

5

How soon before surgery should an antiplatelet drug such as clopidogrel be discontinued?

5 days before surgery

What does "D5W" mean

5% dextrose in sterile water (% grams per 100mL)

The fibrin strands form an insoluble web causing blood clotting. How long does this normally take to occur

6 minutes

Match the following cancer and cancer treatment terms with the correct definition: GROWTH FRACTION

A MEASURE OF THE NUMBER OF CELLS UNDERGOING MITOSIS IN A TISSUE

Match the stomach acid reducing medication with its classification: ALUMINUM HYDROXIDE

ANTACID

Match the Classification of drug with its use for specific conditions:UNSTABLE ANGINA

ANTIPLATELET

Match the Classification of drug with its use for specific conditions:VASCULAR BYPASS

ANTIPLATELET

Your patient has asked you how to use the cetirizine dissolving tablet. What would you include in your teaching?

Allow that table to dissolve in your mouth without chewing and swallow several times as the tablet dissolves.

The newest class of drugs for type 2 diabetes is called sodium-glucose co-transporter 2 (SGLT-2) inhibitors. What is an SGLT-2?

An enzyme in the kidney tubule that causes glucose to be reabsorbed from the urine.

Special administration instructions of antacids include: (Choose all that apply)

Antacids can cause constipation Taking antacids 2 hours before or 2 hours after other drugs If a bowel obstruction is suspected, do not take antacids.

If a nurse suspects a patient has a bowel impaction, which drugs should not be administered? Check all that apply.

Anti-diarrheals like Bismuth salts (Pepto Bismol) Antacids like Aluminum Hydroxide

Colchicine is prescribed for its:

Anti-inflammatory properties

Patients should be educated that St. John's Wort can affect patients taking:

Antibiotics Birth control pills Warfarin Digoxin Anti-depressants like SSRIs Seizure medications like carbmazepine

A patient on your watch became very agitated and confused after medications you administered. Which of the following is the best explanation for his overall clinical presentation?

Anticholinergic effects of the promethazine precipitated delirium, and lorazepam had a paradoxical effect.

A patient starts vomiting prior to chemotherapy. What is the probably cause?

Anticipatory nausea and vomiting because the patient expects and unpleasant experience.

A patient is being treated for a thromboembolic disorder. If the goal is to prevent clot formation, the nurse anticipates the patient will be treated with which classification of drug? check all that apply.

Anticoagulants Antiplatelet agents

Ipratropium inhaler is contraindicated for patients who

Are allergic to soybeans or peanuts

Which of the following actions by the nurse is most important when caring for a client with renal disease who has an order for furosemide?

Assess urine output and renal laboratory values for signs and nephrotoxicity

For which patient;s conditions i timolol eye drops, a beta blocker, contraindicated

Asthma COPD Heart block Slow heart rate

The child most likely to receive risperidone to manage symptoms is one diagnosed with

Autistic spectrum disorder (ASD)

What is beneficial in reducing the risk of Reye syndrome?

Avoidance of aspirin to treat fever associated with influenza

Match the drug with its classification: ALBUTEROL

BETA-ADRENERGIC AGONIST

Match the stomach medication with its action: OMEPRAZOLE

Blocks hydrogen ion production in the stomach which drops the amount of acid secretion

How are drugs delivered directly to the respiratory system?

By aerosol

When is the preferred time to give ondansetron (Zofran) for a client receiving antineoplastic therapy?

Prior to the antineoplastic therapy

When giving warfarin to a patient, which lab values are important for the nurse to review prior to administration?

Prothrombin time (PT) International normalized ratio (INR)

Intranasal phenylephrine causes:

Rebound Congestion

What is a common side effect of vancomycin?

Red-man syndrome

What is the mechanism of action of metformin? Check all that apply

Reduces insulin resistance Decreases the hepatic production of glucose

Precautions or contraindications for patients on H2 blockers include. (Choose all that apply).

Renal disease Blood dyscrasias such as anemia, neutropenia or thrombocytopenia hepatic disease (all but bowel obstruction)

To monitor for superinfection what patient teaching should you provide?

Report a fever that does not diminish within 3 days

What teaching should you provide to a patient taking levofloxacin?

Report heel pain

Why would a patient be taking hydroxychloroquine?

Rheumatoid arthritis (yes, hydroxychloroquines is an antirheumatic drug used to slow the disease and enhance the quality of life for those with RA.

When a patient is started on cangliflozin what is critical to teach the client? Check all that apply.

Risk of and signs of hypoglycemia To call your health care provider if you have s/s of a urinary tract infection

For which one of the following types of drugs is the consumption of grapefruit juice contraindicated?

Second-generation antipsychotics

A patient with depression has taken a selective serotonin reuptake inhibitors (SSRIs) for 1 month. The nurse should use direct questions to evaluate which potential side effect?

Sexual dysfunction

Signs and symptoms of lithium toxicity include: (select all that apply)

Slurred speech Drowsiness

Which of the following classes of hypoglycemics are more likely to cause hypoglycemia? Check all that apply.

Sufonylureas Meglitinides

Match with drug class: CHLOROTHIAZIDE

THIAZIDE DIURECTIC

A nurse notes that a client with schizophrenia and receiving an antipsychotic medication is having uncontrolled movement of the lips and tongue. The nurse determines that the client is experiencing?

Tardive dyskinesia

Metoclopramide (Reglan) should be used short term only because it may cause a serious movement disorder that is not reversible. What disorder is that?

Tardive dyskinesia

A patient with Type 1 DM complains of feeling sweating and shaky at 6 pm, right before dinner. His blood sugar registers 58 on the glucometer. This can be attributed to the effects of which dose of insulin?

The 12 noon dose of lantus insulin (lantus peaks 6 hours after administration)

What is the priority goal of antiemetic therapy?

The complete prevention of chemotherapy induced nausea and vomiting

Why is it recommended that all children receive the HPV vaccination?

The human papilloma virus is associated with the development of cervical cancer in women and males can be carriers of the virus.

Which of the following is an advantage of insulin analogs over older forms of insulin?

They may have a more prolonged duration of action or more rapid onset of action

After 2 weeks of using latanoprost for open-angle glaucoma, the patient complains that he has irritation, stinging and redness in his eyes. Education the nurse would provide to the patient would include:

This is a normal response during the first month of therapy. Use the drops at bedtime so you are not feeling that irritation as much

Calcitonin is a hormone secreted by the............when serum calcium is elevated

Thyroid gland

The client receives diphenhydramine (Benadryl) to control allergic symptoms. Which common symptom does the nurse teach the client to report to the physician?

Urinary hesitancy

While receiving chemotherapy sexually active male and female patients should be instructed to:

Use reliable contraception during and for 3 to 6 months following treatment

While teaching a client how to administer a liquid medication, the nurse will instruct the client to use which method for measuring the liquid?

Use the dropper enclosed in the package the medicine come in

A patient who has a new prescription of an ipratropium inhaler is asking the nurse about when to use the inhaler. The best instruction by the nurse is

Use the ipratropium regularly at the same times each day (this will be either 3 or 4 times a day)

Match the common adverse effects with the correct cause: EXTRAVASATION

VESICANTS ESCAPE FROM AN ARTERY OR A VEIN DURING INFUSION

Match the common adverse effects with the correct cause: EMETOGENIC

VOMITING CENTER IN THE MEDULLA IS TRIGGERED

The physician discusses the appropriateness of prescribing a serotonin/norepinephrine reuptake inhibitor (SNRI) for a patient. Which of the following medications are in this category?

Venlafaxine and duloxetine

Milirone is a medication used to treat heart failure. The patient knows when taking this medication it may put him/her at risk for:

Ventricular dysrhythmias

A patients INR is 6. What drug can the nurse expect to give to revers a high level of Warfarin in the blood?

Vitamin K

taking increased amounts of which supplements can cause build-up in the fatty tissues of the body? (Select all that apply)

Vitamin K Vitamin D Vitamin E Vitamin A

Which of the following drugs may be toxic in elderly patients because serum albumin levels decrease with age?

Warfarin

Patients prior to surgery should be asked about their use of:

Warfarin Vitamin E

The nurse is preparing to administer a newly ordered statin drug to a patient and is reviewing the patient's list of current medications. Which medications may cause an interaction with the statin drug? Select all that apply

Warfarin Erythromycin

When a patient requires anticoagulation therapy they are often prescribed heparin and warfarin concurrently. Why is this done?

Warfarin takes several days to achieve optimum effect

A patient's absolute neutrophil count is 1000/mm3. To prevent complications the most important thing a nurse will do is:

Wash hands before entering the room

What are the most common adverse effects of thiazolidinediones (TZDs)? Check all that apply

Weight gain Fluid retention Headache

A client with schizophrenia has been started on medication therapy with clozapine. A nurse assess the results of which laboratory study to monitor for adverse effect related to this medication?

White blood cell

Thrombolytics are used in the treatment of MI's. When should they be administered?

Within 30 minute of arrival to the hospital

Uses for tretinoin, a natural derivative of vitamin include: select all that apply

Wrinkle reducer Acne treatment

While teaching a client about Zafirlukast's side effects, the nurse will instruct the doctor to call for which of the following?

Yellowing of the eyes/skin

A nurse counsels a patient who has been prescribed a monoamine oxidase inhibitor (MAOI) to treat depression to avoid foods containing tyramine. Which of the following foods should the patient avoid? (select all that apply)

Yogurt Pepperoni Tofu

The nurse is caring for a patient with neutropenia. He wants to know what that means and why he has it. What is the best instruction by the nurse?

You have a decreased number of neutrophils because the bone marrow is affected by the chemotherapy

Calcitonin, used to help put calcium back in the bones ("calcitonin tones the bones"!), is primarily administered through the intranasal inhalation route. What side patient teaching should be done for the patient for this drug?

Your nose may become irritated

When alcohol is taken with sulfonylureas such as glimepiride, glipizide, or glyburide some patients experience:

a disulfiram-like reaction

A patient comes to the clinic complaining of angina. While getting a history from the patient the nurse takes note that:

a patient taking Sildenafil should not be given nitrates for angina

Laboratory studies related to heparin therapy include

aPtt

Trimethoprim and sulfamethoxazole should not be given to infants under two months old. This is because of what possible complication?

acute bilirubin encephalopathy

MEglitinides such as nateglinide need to be taken

at or before each meal

Match the Category with its fluid replacement agent: PACKED RBC

blood prdouct

Atropine is used to treat what type of heart dysrhythmia

bradycardia

A patient with asthma who uses albuterol should avoid which of the following

caffeine

A patient with severe kidney disease is started on lithium by a new resident. As the nurse on duty your best initial response would be to:

contact the resident to remind him that the client has kidney disease and lithium is contraindicated

When assessing a patient taking enalapril for the first time the nurse knows to check the patient for:

decrease in blood pressure

Patients can experience relief of heart failure symptoms using vasodilation to

decrease the cardiac workload

Alendronate is prescribed for?

decreasing bone resorption

People who eat large amounts of carrots or carrot juice may notice:

discoloration of the skin

As an adult how often should a tetanus immunization be given?

every 10 years

Select the accurate information about the pharmacokinetics of lithium

excreted by the kidneys

true or false. Zafirlukast works quickly and may be used to treat an asthma attack that has already begun

false

The most important food for a patient taking anticoagulants to avoid is

garlic

Dextroamphetamine is prescribed to an 8-year-old child for the treatment of ADHD. The nurse plans to monitor for which of the following during the medication therapy?

height and weight (Methylphenidate and other stimulants may cause slow growth and reduce appetite. the nurse will need to keep track of the client's height and weight to make sure that there is a normal growth and developments)

For which of the following is protamine sulfate used as an atagonist

heparin

In reviewing the history of a newly admitted cardiac patient, the nurse knows that the patient would have a contraindication to antilipemic therapy if which condition is present

liver disease

Thrombolytic drugs are used to

lyse a thrombus.

Match the antibiotic with the class that it belongs to: AZITHROMYCIN

macrolide

For what condition of the eye are timolol eye drops frequently prescribed?

open-angle glaucoma

A child's mother tells you, "the doctor has been prescribed our daughter digoxin to take at home. are there any special things we should do?" What patient teaching is needed.

prior to administration take an apical pulse. Do not give the medication if the pulse is below 100 BPM

Anticonvulsants frequently used as mood stabilizers for the treatment o mania and bipolar spectrum disorders because they:

reduce synaptic kindling

Metoprolol is a drug of choice when treating heart failure because it:

reduces long-term effects of the disease

A patient is prescribed nifedipine for high blood pressure. The nurse knows teaching was successful when the patient states this medication works by:

relaxing arterial smooth muscle.

A newly admitted client has started taking bupropion. The nurse understands which of the following side effects that would indicate na overdose of the medication

seizure

The therapeutic effects of imipramine develop over ..............

several weeks

Beta-adrenergic blockers reduce symptoms of heart failure by:

slowing the heart rate and lowering blood pressure

Maintenance of a therapeutic serum level of lithium is dependent on adequate serum levels of:

sodium

What solute is the greatest contributor to osmolality due to its abundance in the most body fluids?

sodium

A 19-year-old female client with depression who has been taking amitriptyline for three weeks returns to the clinic for a follow-up. The nurse anticipates the possibility of observing which of the following symptoms

suicidal thoughts

A 15-year-old with a 6-month history of moderate to severe depression started anti-depressant therapy with Sertraline two weeks ago. The nurse will teach the family that he is most at risk during the first 3 weeks of medication therapy for

suicidality

An acutely manic patient is started on divalproex. The experience nurse understands valproate acid can be measured:

through a serum blood level

Psychopharmaceutical agents are prescribed for individuals with substance use disorders for all of the following reasons except

to elevate the patient's mood in a manner comparable to the action of the misused substance (psychopharmaceutical agents are not prescribed to mimic abused substances)

A client taking lithium carbonate started complaining of nausea, vomiting, diarrhea, drowsiness, muscle weakness, tremor, blurred vision and ringing in the ears. The lithium serum level is 2 mEq/L. The nurse interprets this value as:

toxic level

Pharmacotherapy to terminate migraines generally begins with acetaminophen or NSAIDs. If the OTC or milder prescription analgesics are unable to abort the migraine, which classification is frequently the drug of choice

triptans

All of the following are actions of phenylephrine EXCEPT:

DECREASES BLOOD PRESSURE (INCREASES NOT DECREASES)

Mach the drug to the mechanism of action: PIOGLITAZONE

DECREASES INSULIN PRIMARILY AND INHIBITS HEPATOGLUCONEOGENESIS SECONDARILY

The patient tells the nurse that his wife insists he takes herbal remedies while he is on chemotherapy. The nurse will:

Determine which herbal remedies he is using and document them in the patient's chart

Nurse Trini knows that the following drugs have been known to be effective in treating OCD

Fluvoxamine and clomipramine

Neural tube formation in the developing fetus relies upon which vitamin?

Folic acid

With which lab value would the nurse expect to give IV potassium chloride to treat a patient's potassium level?

For a blood potassium level of 2.8

Because cromolyn inhalers used for prophylaxis of asthma have a short half life, how frequently must it be inhaled?

Four to six times per day

A patient with chronic hyperacidity of the stomach takes aluminum hydroxide on a regular basis. The patient presents with complaints of increasing weakness. What may be the cause of this increasing weakness?

hypophosphatemia

A patient is receiving an IV infusion of milrinone. The nurse knows a side effect of the medication is:

hypotension

Patient teaching for Lorazepam includes

if a dose of lorazepam is missed, do not double up on missed dose

Nortriptyline was prescribed for a 68-year-old patient diagnosed with depression and insomnia. Benefits specific to use of nortriptyline would include:

improved sleep pattern

Methylphenidate is a CNS stimulant and is the drug of choice for individuals with attention- deficit/hyperactivity disorder (ADHD). The desired effects of methylphenidate may be noted as:

increased attention span and concentration.

What is the anticipated effect of drug distribution on volume in elderly patients who have increased body fat?

increased volume of distribution for highly lipophilic drugs

Phosphodiesterase inhibitors help with heart failure by:

increasing the force of myocardial contraction

A male patient who is taking the second-generation antipsychotic reports to the nurse that he has observed a clear fluid leaking from his right nipple for the past month> The nurse best initial intervention is

inform the patient that this could be a side effect of the medication

The nurse is aware that the mechanism of action of anticoagulant drug is to

inhibit clotting factors to prevent clot formation

Patient taking tricyclic antidepressants (TCAs) exhibit more side effects when patients taking SSRIs because TCAs

inhibit reuptake of norepinephrine and serotonin

When taking digoxin the patient should:

know how to check a radial pulse

Select the most realistic short-term goal for the care of a patient with mild Alzheimer's disease who take donepezil. The patient will:

maintain present cognitive ability.

Patients with............are at risk of developing exaggerated symptoms of their disorder during early therapy with Benztropine

mental illness

This medication increases heart contractility, decreases pulmonary vascular resistance, and decrease afterload by vasodilation

milirone

After taking the few doses of doxazosin, patient instructions include which of the following?

monitor for the possibility of syncope 2-6 hours after taking the medication

A patient diagnosed with schizophrenia has taken thiothixene 5mg/day for 4 years with good symptom control. Today, the patient is admitted with paranoia and auditory hallucinations. The nurse should determine if the patient's symptoms are related to:

not taking the drug as prescribed

Which of the following interventions is the most important intervention when administering a plasma volume expander to a patient?

observe for signs of fluid overload

What drug is used as a weight control medication by inhibiting lipase?

orlistat

Monoamine oxidase inhibitors (MAOIs) can cause numbness, prickling or tingling feelings, which are called

paresthesias

Adenosine is primarily used to treat what type of heart dysrhythmia

paroxysmal supra ventricular tachycardia (PSVT)

Thiazolidnediones (TZDs) include which of the following medications? Check all that apply

pioglitazone rosiglitazone

Fibrinolysis is the result of

plasmin digesting the fibrin strands of clots

Anticoagulants are used to

prevent the formation of blood clots

A patient is prescribed enalapril for HTN. The nurse knows teaching was successful when the patient states this medication works by:

preventing vasoconstriction

Individuals prescribed antibiotics may also be prescribed which supplement?

probiotics

Which of the following has a high risk of toxic effects due to age-related decreases in first-pass (usually hepatic) metabolism?

propranolol

Which over-the-counter drug will the patient have to show her ID to purchase and be distributed from behind the counter, due to tighter restrictions on people wanting to buy in bulk for the raw ingredient in it?

pseudoephedrine

When administering any antidysrhythmic drug, it is important to monitor:

pulse

Patients can experience relief of heart failure symptoms by using furosemide. Its mechanism of action includes:

reducing fluid volume

Because hepatic metabolism of many drugs decrease with age, drugs metabolized in the liver during phase 1 will be cleared....

slower

A patient started diazepam (Valium) 5 mg twice daily 6 months ago. Now, the patient requires 10 mg to achieve the same effect. This phenomenon results form:

tolerance

Administering risperidone with levodopa is contraindicated.

true

Itching is a side effect of morphine administration. It does not necessarily indicate that a patient has an allergy to morphine

true

T/F Teaching for parents of children with lice may include normalizing a lice infestation, as it is common for school-aged children to have.

true

When takin the history of a child hospitalized with Reye syndrome, the nurse should not be surprised that a week ago the child had recovered from:

varicella

A nurse is preparing to give an IM injection to a patient that is 6 months old. What injection site would be best?

vastus lateralis

How often should the plastic mouthpiece of an inhaler be washed?

weekly

Hair follicles are damaged by many chemotherapeutic agents. Hair loss usually begins:

within 1 to 2 weeks after the first treatment

A client with depression is taking phenelzine. The nurse advises the client to avoid consuming which foods while taking the medication

yogurt

Why are inhaled corticosteroids such as beclomethasone the preferred therapy for preventing asthma attacks?

Inhaled corticosteroids suppress inflammation without producing major adverse effects

A patient receiving Alendronate (Fosamax) to treat osteoporosis becomes discouraged due to perceiving "not feeling stronger bones yet" and states that he is going to stop taking the medication. Teaching for this patient should include: (mark all that apply)

Instructions for proper administration (e.g., taken with water) Information regarding peak time for drug effectiveness, as this medication might take 1-3 months to reach full effect

Which of the following types of insulin is long acting?

Insulin glargine

Vegans, vegetarians, and women with heavy menstrual bleeding may be encouraged by their doctors to supplement with:

Iron

Which of the following is generally true of medications used to treat anxiety disorders?

Just as with depression, the most common medications prescribed are serotonin reuptake inhibitors (SSRIs) and selective norepinephrine reuptake inhibitors (SNRIs)

In an acute trauma situation the preferred replacement fluid is?

Lactated Ringers

Match the following Pharmacotherapy for allergic rhinitis CROMOLYN INTRANASAL

MAST CELL STABILIZER

Match the interaction of sodium bicarbonate with the following medications: ASPIRIN AND OTHER SALICYLATES

MAY CAUSE THIS CLASS TO BE EXCRETED FASTER, SO EFFECTIVENESS WOULD BE SHORT FOR PAIN OR ANTI-PYRETIC CONTROL

Match the interaction of sodium bicarbonate with the following medications: LITHIUM

MAY CAUSE THIS DRUG TO BE EXCRETED FASTER, SO THE THERAPEUTIC LEVELS WON'T STAY IN RANGE

Match the following Pharmacotherapy for coughs: ACETYL CYSTEINE

MUCOLYTIC

Match the medications listed below with the complication of chemotherapy for which it is prescribed: COCKTAIL WITH LIDOCAINE, DIPHENHYDRAMINE, AND MAALOX

MUCOSITIS

A patient is going into shock due to complications of heart failure. The doctor prescribed dopamine. The nurse knows this medication can be used as a short term treatment because it:

Maintains blood flow to vital organs

Patients at risk for low levels of Vitamin D

May supplement with oral vitamin D Can increase their levels by spending time in the sunlight May benefit from milk with added Vitamin D (Everything but should avoid exposure to sunlight)

There are several leukotriene receptor antagonists used to treat asthma. Identify which of the following are leukotriene modifiers/inhibitors. Choose all that apply

Montelukast Zafirlukast

Common manifestations of decreased magnesium levels in the body is/are:

Muscle spasms and leg cramps

Symptoms of hypocalcemia are: Choose all that apply.

Muscle twitching and tremors Cardiac dysrhythmias numbness and tingling of the extremities abdominal cramping

What assessment findings in a patient taking calcitriol should the nurse recognize and report to the provider immediately?

Muscle twitching, numbness, and tingling of the extremities

Match the antibiotic with the class that it belongs to: OXYMETAZOLINE

NASAL DECONGESTANT

Match the following Pharmacotherapy for allergic rhinitis: OXYMETAZOLINE INTRANASAL

NASAL DECONGESTANT

Your client has been taking cetirizine twice a day and is now concerned because he realizes it is only to be taken once a day. For which signs and symptoms will you send him for emergency medical attention?

Nervousness, restlessness followed by extreme lethargy.

Match the following medications with the most accurate drug classification: NALOXONE (NARCAN)

OPIOD ANTAGONIST

What principle of physics do colloids use to move water across the vascular membrane?

Oncotic pressure

The patient is concerned about having chemotherapy induced nausea and vomiting (CINV). The nurse will instruct the patient to use which of the following medications prior to chemotherapy to prevent CINV?

Ondansetron

Methylxanthines such as theophylline are administered by which routes? Check all that apply.

Oral Intravenous

A post-operative patient is being taught about pain management. The nurse will teach the patient which of the following?

Our goal is pain relief, rather than pain elimination

Match the medication with its classification: PANCRELIPASE

PANCREATIC ENZYME REPLACEMENT

Match with drug class: SPIRONOLACTONE

POTASSIUM SPARING DIURETIC

Match the following cancer and cancer treatment terms with the correct definition: GRADING

PROCESS THAT EXAMINES CANCER CELLS UNDER A MICROSCOPE

Mach the drug to the mechanism of action: INSULIN

PROMOTES THE CELLULAR UPTAKE OF GLUCOSE, AMINO ACIDS, AND POTASSIUM

The blood bank is going to separate out component parts of blood. Choose which part of the blood matches which description: GLOBULINS

PROTEINS IN THE BLOOD, WHICH WHEN GIVEN CAN HELP THE IMMUNE SYSTEM FIGHT. THEIR ARE ACTUALLY 4 KINDS OF THESE

A priority nursing care for a patient being started on Amiodarone is:

Place patient on continuous EKG monitoring

A newborn is give IV antibiotics. What assessment parameter addresses the newborns ability to distribute the antibiotic throughout the body?

Plasma protein levels

What is the goal of chemoprophylaxis?

Prevent cancer in patients at high risk for developing tumors.

Raloxifene is a selective estrogen receptor modulator (SERM.) It is primarily used for the prevention of osteoporosis in postmenopausal women. Other uses include which of the following:

Prevention of vertebral fractures caused by osteoporosis of the spine

A class of incretin enhancers known as DPP-4 inhibitors do which of the following?

Prevents the breakdown of incretins

How do SGLT-2 inhibitors work?

Prevents the kidneys from reabsorbing glucose back into the blood.

The client receives beclomethasone (Beconase) intranasal as treatment for stuffiness caused by allergic rhinitis. He asks the nurse if this drug is safe because it is a glucocorticoid. What is the best response by the nurse? "intranasal glucocorticoids..." (Check all that apply)

Produce almost no serious adverse effects when taken intranasal, but steroids taken systemically can have serious adverse effects Are safe if you don't swallow during administration. So, don't swallow after you inhale it.

What physiologic change associated with normal aging may affect geriatric patient's clinical response to medications

Prolonged half-life of lipid soluble drugs, due to a higher relative composition of body fat.

Patient teaching of Lorazepam should include the following: (mark all that apply)

Prolonged high-dose therapy may lead to psychological or physical dependence Common adverse effects of Lorazepam are drowsiness and sedation, which may decrease with time

A patient with Type I diabetes asks the nurse why propranolol was stopped by his endocrinologist? The nurse replies:

Propanolol may mask the symptoms of hypoglycemia.

A patient is prescribed Sildenafil for pulmonary hypertension associated with heart failure. The patient says I thought this medication was for erectile dysfunction. Why am I taking it? What is the best response by the nurse?

Sildenafil relaxes msucles and increases blood flow to pulmonary arteries

A patient is in acidosis secondary to DKA. What medication will best help to correct the acidosis

Sodium Bicarbonate

Procainamide is what type of antidysrhythmic?

Sodium Channel Blocker

A patient has metabolic acidosis. Which IV fluid would the RN expect to begin?

Sodium bicarbonate

A client has been prescribed Hycomine Compound, an opioid combination drug for control of cough. This drug contains hydrocodone, phenyephrine, chlorpheniramine, and acetaminophen. Which instructions are a priority part of medication education?

Take this drug exactly as directed

Teaching instructions for patient taking aspirin should include: (select all that apply)

Take with food or milk Avoid alcohol

The patient is taking promethazine hydrochloride IV for nausea. Which location of the body should be monitored while this medication is administered?

The IV administration site

When is isotretinoin indicated for the treatment of acne during adolescence?

The acne has not responded to other treatments

For whom is insulin replacement therapy required? Check all that apply

Those with type 2 diabetes who are unable to manage their blood glucose levels with diet, exercise and oral antibiotic drugs patients with type 1 diabetes mellitus

A patient has been started on ticlopidine (Ticlid) after a myocardial infarction and stent placement. How should the nurse explain the action of this medication to the patient? Select all that apply.

Ticlid will change the way your platelets work their entire lives ticlid decreases your blood's ability to clot ticlid will make the platelets in your blood less sticky

Phenylephrine (Neo-synephrine) is a nasal decongestant. Why would it be used as a treatment for nose bleeds?

To constrict small blood vessels

Corticosteroids are given to patients with asthma for which of the following reasons? Check all that apply

To dampen the activation of inflammation and increase production of anti-inflammatory mediators To reduce the bronchial hyper-responsiveness to allergens

Why might a child be prescribed topical permethrin?

To eliminate hatched lice

What is the role of calcium in cardiac smooth muscles?

To facilitate contraction

To whom should normal saline be contraindicated? (Choose all that apply)

To hypernatremic patients To dialysis patients To CHF patients

When teaching H. pylori, the nurse will recognize that using two or more antibiotics is indicated for what reason?

To increase effectiveness and lower the potential for bacterial resistance

When a client is routinely taking NSAIDS, a nurse may suggest a Guaiac stool test be ordered. Why would the nurse feel this is necessary?

To monitor for GI bleeding

What is the purposed of adjuvant chemotherapy?

To rid the body of cancerous cells after surgery or radiation

A patient currently being treated for depression with daily sertraline (Zoloft), an SSRI antidepressant, is coming to your unit postoperatively. What medication prescription should the nurse note as contraindicated in patients?

Tramadol (Ultram) 50mg PO BID PRN for pain

Nitroglycerine can be administered in all the following ways EXCEPT?

Transrectally

Because children may develop a low-grade fever after an immunization, counsel parents they may need to give acetaminophen or ibuprofen for a fever higher than 38.4 C (101 F). (T/F)

True

Bupropion can be used for smoking cessation

True

Chemotherapy drugs may be used for palliation

True

Clinicians will frequently under prescribe medications because of increased concern about adverse effects (T/F)

True

Cyclobenzaprine should be used only in adults (T/F)

True

Ethosuximide is the preferred drug for managing absence seizures (T/F)

True

IV potassium chloride replacement needs to be given slowly since it has a tendency to be caustic to the veins (T/F)

True

Low doses of Atropine decreases sweating, salivation, and respiratory secretions (T/F)

True

T/F Hydroxychloroquine would be prescribed for a patient with rheumatoid arthritis early in the disease process

True

T/F Pentazocine exerts mixed opioid agonist-antagonist effects

True

T/F Taking Calcitriol (vitamin D) can cause hypercalcemia.

True

T/F: Onabotulinumtoxin A (Botox) was approved in 2010 for the treatment of chronic migraines if OTC and otehr prescription medications were not successful

True

The medications that are given to kill the cancer may cause a secondary cancer to develop elsewhere in the body

True

True or False. Combivent is a mixture of iprtropium and albuterol in a single meter dose inhaler

True

True or False. Several anti-nausea medications are available OTC

True

Within the healthcare industry, the effectiveness and strength of all other opioids are described in comparison to morphine's effectiveness (T/F)

True

Cyclobenzaprine should be used with caution in adults over age 65 (T/F)

True (this population is more likely to experience confusion, hallucinations, ad adverse cardiac events from this drug.)

Which of the following nursing assessment criteria would be appropriate to do when administering opioid agonists? Choose all that apply

Vital signs with an emphasis on respiratory rate Level of consciousness Bowel Sounds

Patients who have had gastric bypass surgery are at risk for which vitamin deficiency?

Vitamin B12

Which vitamin may be decreased by chronic use of metformin?

Vitamin B12

A patient has been an alcoholic for 5 years and enters the hospital system. A pregnant women is prescribed a supplement for the health of her unborn child's development. What vitamin would you expect to see on both of their medication administration records?

Vitamin B9 (folic acid promotes fetal growth as well as supporting alcohol related deficiencies)

People taking an oral iron supplement should take it with which foods to increase absorption? (Select all that apply)

Vitamin C Orange Juice

A nurse working night shift may be at risk for a deficiency of which vitamin?

Vitamin D

Which supplement assists calcium to build strong build strong bones?

Vitamin D

Patients taking warfarin should be cautious when taking

Vitamin E

A client who received chemotherapy 7 days ago calls and reports a temperature of 101.2 but denies chills, cough, or sore throat. What will you tell the client?

You will need to be seen right away. You are at increased risk for a bacterial infection.

Hydroxychloroquine overdose in children may be life threatening. Treatment for overdose may include: (mark all that apply)

Antidysrhythmics Anticonvulsants Vasopressors

Possible nursing diagnoses for patients receiving Diazepam include: (mark all that apply)

Anxiety Impaired physical mobility Risk for Injury

Diphenhydramine can be used for the following conditions (select all that apply)

Anxiety and insomnia anaphylaxis allergic rhinitis Parkinson's disease

What food should you not combine with Azithromycin?

Apple Juice

A patient is receiving potassium chloride IV for a serum potassium of 3.1 mEq/L. Which severe side effect of administering the potassium too quickly must the nurse be aware of?

Arrythmias leading to cardiac arrest

A female patient is taking ampicillin. What is important to tell her about the prescription?

Ask her about oral contraceptive use.

If a patient is taking Bismuth salts, which medication on the patient's medication list would alert the nurse to an issue?

Aspirin

The nurse will need to take a thorough medication history if the patient is taking warfarin and which of the following drugs? select all that apply

Aspirin Ibuprofen Acetaminophen

People taking warfarin should be taught: (CHOOSE ALL THAT APPLY)

Avoid cranberry juice Avoid alcohol Avoid ginger, garlic, ginseng, ginko, and glucosamine Broccoli and cabbage can decrease the effects of warfarin (everything but spinach, kale, and brussel sprouts answer)

Mr. Proboscis uses a nasal spray of fluticasone propionate (Flonase), a corticosteroid, for his perennial allergic rhinitis. In order for him to avoid the potential adverse side effects associated with its medication, which of the following instructions will you include in your teaching?

Avoid swallowing the medication

Match the drug with its classification: SALMETEROL

BETA-ADRENERGIC AGONIST

Mach the drug to the mechanism of action: CANAGLIFLOZIN

BLOCKS THE KIDNEY REABSORPTION OF GLUCOSE

Match the Category with its fluid replacement agent: FRESH FROZEN PLASMA

BLOOD PRODUCT

Match the Category with its fluid replacement agent: WHOLE BLOOD

BLOOD PRODUCT

Match the following classes of drugs with how they are used to relieve symptoms and avoid complications of COPD: BETA 2 AGONISTS

BRONCHODILATION

The patient is currently taking warfarin (Coumadin) and become pregnant. The physician changes her medication to enoxaparin (Lovenox). She asks the nurse. "why did the doctor change my medication?" what is the best rationale by the nurse?

Because enoxaparin (Lovenox) does not cross the placenta

When planning pain management for a patient who is experiencing acute pain, when is the best time to administer an opioid agonist, such as hydromorphone?

Before an exacerbation of the pain

When is the best and most effective time for oral antiemetic therapy if a patient is receiving chemotherapy?

Before starting chemotherapy with high emetic potential

A patient has taken clonazepam for years to manage panic attacks but impulsively stopped the drug. Thirty hours later, the patient comes to the emergency room in distress. What is the nurse's priority action?

Begin seizure precautions

A patient who was prescribed albuterol extended release tabs one month ago called the office complaining of chest pain and fast, pounding or uneven heart beats. After telling him to call 9-1-1 you discuss what may have happened with the healthcare provider. What is the likely cause which you should be aware of for this patient?

Beta 1 receptor activation in the heart

Which of the following will enhance bronchodilation?

Beta 2 adrenergic agonists

What type of antidysrhythmic is propranolol?

Beta Blocker

Which of the following are medications for migraine prophylaxis?

Beta adrenergic blockers

For a patient with COPD which of the following drugs should be avoided? Check all that apply.

Beta-adrenergic antagonist Opioids

Which of the following are medications for migraine prophylaxis?

Beta-adrenergic blockers

A patient taking Warfarin for atrial fibrillation is being checked for a therapeutic level of the drug in his system. His INR is expected to be:

Between 2-3

In addition to multiple antibiotics, what else would the nurse anticipate will be added to the medication regimen to treat H. pylori? (Choose all that apply)

Bismuth compounds proton pump inhibitor

Hypovolemic shock an be triggered by which of the following conditions: (choose all that apply)

Bleeding from trauma Excessive or prolonged diarrhea Excessive sweating

Match the stomach medication with its action: RANITIDINE HYROXIDE

Blocks the histamine sites in the stomach which reduces the volume of acid secreted

Allopurinol is given for what purpose when treating gout

Blocks xanthine oxidase which keeps the body from forming uric acid crystals

The nurse knows that all of the following are risk factors for peptic ulcer disease except:

Blood group A

A nurse will expect which symptom or occurrence when giving Dextran 40?

Blood pressure risking from 110/62 to 145/81

Which of the following nursing assessment criteria would be appropriate to do when administering opioid agonists such as oxycodone, fentanyl and morphine? Choose all that apply

Bowel sounds Vital signs with an emphasis on respiratory rate Level of consciousness

Match the medication with its classification: PSYLLIUM MUCILLOID

Bulk laxative

The FDA-approved indications for fluoxetine include: (select all that apply)

Bullimia nervosa Obsessive-compulsive disorder Major depressive disorder

Calcitriol can be obtained through which of the following routes?

By spending time with skin exposed to the sun Capsule Tablet

Theophylline is currently used for the long term oral prophylaxis of asthma that is unresponsive to beta agonists or inhaled corticosteroids. It is chemically similar to which of the following common drugs?

Caffeine

What vitamin is also known as Vitamin D?

Calcitriol

What is a LR composed of

Calcium Sterile water potassium sodium lactate sodium chloride

Patients concerned about losing bone density can be encouraged to talk to their doctor about supplementing with:

Calcium Vitamin D

Verapamil is what kind of antidysrhythmic?

Calcium Channel Blocker

What class of drug is Diltiazem?

Calcium Channel Blocker

With age, small bowel surface area decreases, gastric emptying slows, and gastric pH increases. Changes in drug absorption in the elderly tend to be clinically inconsequential for most drugs. which of the following is less effectively absorbed in this environment

Calcium carbonate

Nursing practice when administering antidysrhythmics include all except?

Calling the Doctor prior to administration

Why are meglitinides better tolerated than sulfonylureas? Check all that apply.

Cause less hypoglycemia Cause less weight gain

What is the mechanism of action for triptans, such as sumatriptan

Cause vasoconstriction of cranial arteries

Conditions that my be helped by Dantrolene include:

Cerebal palsy Spinal cord injury

When educating a patient on laxative use the nurse knows it is important to include what teaching?

Chronic laxative use can result in fluid loss and hyopkalemia

What is the concern in a patient taking diphenhydramine (Benadryl) and monoamine oxidase inhibitors (MAOIs) for depression?

Client may develop a hypertensive crisis

Which of the following symptoms is a sing of mild lithium toxicity:

Coarse hand tremor and lethargy

Match the Category with its fluid replacement agent: ALBUMIN

Colloid

What teaching should you provide to a patient on long term antibiotics?

Consume dairy products with live action cultures

Administration alerts for physostigmine include (select all that apply)

Continuous infusions should never be used Monitor BP, Pulse, & RR Administer slowly over 5 min

When teaching a patient about adverse side effects of digoxin. The nurse should include which of the following?

Anorexia and nausea

Match the cholesterol lowering agent with patient teaching instructions for taking it: STATINS

TAKE WITH YOUR EVENING MEAL

Match the antibiotic with the class that it belongs to: MINOCYCLINE

TETRACYCLINES

Match the following COPD conditions with the best definitions: EMPHYSEMA

THE BRONCHIOLES LOSE ELASTICITY AND THE AVEOLI DILATE

Match with drug class: METOLAZONE

THIAZIDE DIURETIC

Match with drug class: HYDROCHLOROTHIAZIDE

THIAZIDE DIURETICS

Match drug with the location of action it targets: EARLY DISTAL TUBULE

THIAZIDES

Match the following drug with its indication for use: ATEPLASE, ACTIVASE, TPA

THROBOLYTICS

The blood bank is going to separate out component parts of blood. Choose which part of the blood matches which description: PLATELETS

THROMBOCYTES

Match the medications listed below with the complication of chemotherapy for which it is prescribed: OPRELVEKIN

THROMBOCYTOPENIA

Match the Classification of drug with its use for specific conditions: CVA RELATED TO A THROMBUS FORMATION

THROMBOLYTIC

Match the Classification of drug with its use for specific conditions:CLOT DESCRUCTION

THROMBOLYTIC

A patient has open-angle glaucoma. Choose the medication that would be administered to him for this condition

TImolol

Match the following medications with the proper drug classification: ZOLMITRIPTAN

TRIPTAN

Administration alerts for the antirheumatic drug hydroxychloroquine include: (choose all that apply)

Take at the same time every day Store out of the reach of children

How or when should PPIs be taken?

Take before breakfast on an empty stomach

The waiting time between stopping a monoamine oxidase inhibitor (MAOI) and starting a selective serotonin reuptake inhibitor (SSRI) is which of the folowing

at least 2 weeks

The current recommendation for adults over the age of 65 to receive the follow up 23-valent pneumococcal vaccine (PPSV23) is:

at least one year after the PCV13 vaccine

A tricyclic antidepressant (TCA) is prescribed for a patient newly diagnosed with depression. What information should be included in patient teaching?

"It might be 2 weeks or more before you notice the effects of this medicine."

Which of the following classes of medications included the first oral hypoglycemic available?

Sulfonylureas

Which one of the following drugs to control pain may also cause cardiac ischemia?

Sumatriptan

The emergency department nurse is caring for a client with severe migraine HA. What is the drug of choice to eliminate symptoms?

Sumatriptan (Imitrex)

Which one of the following drugs may cause cardiac ischemia?

Sumatriptan (Imitrex)

The client is prescribed a nasal decongestant spray. What information should the nurse include when educating the client about how to use this medication?

"Limit your use of this spray to no more than 5 days."

What is a common use of Cefazolin?

Surgical prophylaxis

The nurse teaches the client about the difference between oral and nasal decongestants. The nurse evaluates that learning has been effective when the client makes which statement?

"Oral decongestants may cause Hypertension."

An elderly patient has been taking aspirin for a HA lasting three days. The patient states she feels dizzy and has ringing in her ears. The nurse responds correctly:

"This may be associated with taking aspirin. I'll let the doctor know."

The patient is receiving vancomycin to treat sepsis. The nurse would recognize a common adverse reaction would be indicated by which statement?

"What did you say? I didn't hear you."

A client is prescribed an intranasal glucocorticoid for allergic rhinitis and benign nasal polyps. What should the nurse include in client education about this drug?

"Do not eat licorice while taking this drug." (licorice may increase the effects of corticosteroid medications.)

You have just completed teaching your client about the antihistamine cetirizine (Zyrtec). Which statement indicates more teaching is required?

"Drinking alcohol is safe with any of the "new" antihistamines"

All of the following symptoms are very concerning. However which of the following is related to taking Bismuth salts (Peptol Bismol) for diarrhea?

"I am having ringing in my ears, and my hearing has decreased in the last day."

Which of the following statements by a patient on chemotherapy is the most concern to the nurse?

"I am taking my 5 year old granddaughter to the clinic for kindergarten immunizations."

If a patient were receiving IV potassium chloride for potassium replacement, what symptoms would the nurse be concerned about if the patient reported it? (Choose all that apply)

"I'm feeling nauseous."

What response shows that the patient teaching has NOT been successful for the patient taking a laxative for constipation?

"I'm grateful there are products like this laxative for my chronic constipation." (laxatives should not be used chronically)

Which statement by a patient who has been prescribed a nasal decongestant indicates patient teaching has been successful?

"If you take this medication longer than 3 days, I will experience worsening nasal congestion and more mucus."

A patient with peptic ulcer disease will be starting medication therapy. He tells the nurse that he smokes and wonders if that will affect his treatment. Which is the nurse's best response?

"Smoking has been shown to DECREASE the effectiveness of H2 blockers."

An adult says, "I take provastatin for my high cholesterol. It will prevent stroke and heart attack." Select the nurse's most informative response

"Some research indicates that statin drugs may also interfere with development of Alzheimer's disease."

The nurse works in infection control and teaches a class to staff nurses about the ways that resistance to antibiotics can occur. The nurse evaluates that learning has occurred when the nurses make which statement?

"resistance to antibiotics can occur by the common use of them for nosocomial infections."

A nurse is teaching parents of a child recently diagnosed with ADHD who has been prescribed methylphenidate. Which should the nurse include in teaching about the side effects of this medication?

"your child may experience a sense of nervousness."

Tumors are named according to their tissue of origin, therefore most are named with the suffix

-oma

Pancrelipase given to patients with pancreatitis or cystic fibrosis have administration instructions that include (Choose all that apply)

Swallow immediately to reduce changes of irritation to the mouth. Take with plenty of water with every meal or snack Delayed-release capsules should not be crushed to administer.

When a daily dose of Donepezil is missed, it should be taken as soon as the error is discovered. (T/F)

False (missed doses should be skipped and regular schedule returned to the following day)

Taking Psyllium mucilloid is contraindicated in which of the following conditions? (Choose all that apply)

Fecal impaction Intestinal obstruction undiagnosed abdominal pain (everything but open angel glaucoma)

Which patient would be cautioned before taking tretinoin by mouth

Female of childbearing years

Nursing care for patients receiving phenytoin through an enteral tube feeding or NG tube should include (select all that apply):

Residual checks per facility policy A two hour delay between feeding and phenytoin administration Flushing tube with 2-4 oz of water before and after administration

Match the cholesterol lowering agent with patient teaching instructions for taking it: BILE ACID SEQUESTRANTS

TAKE AT LEAST 2 BEFORE MEALS WITH PLENTY OF FLUID

Match the cholesterol lowering agent with patient teaching instructions for taking it: FIBRIC ACID AGENTS

TAKE WITH A MEAL

Life threatening conditions caused by Promethazine (Phenergan) include which of the following? Check all that apply.

Respiratory depression Tardive dyskinesia Neuroleptic malignant syndrome (all but suicidal ideation)

Match the medication with its classification: BISMUTH SALTS

ANTI-DIARRHEAL

Match the medication with its classification: SEMITHICONE

ANTI-FLATULANT

Which individuals may be at risk for low levels of Vitamin D based on their occupations? (Select all that apply)

A compute programmer A nurse who works the day shift A nurse who works the night shift

Match the drug with its classification: IPRATROPIUM

ANTICHOLINERGIC

Which of the following patients should not receive Heparin therapy? (Select all that apply)

A patient with a blood pressure of 195/100 A patient with Hemophilia A A patient with a GI bleed

The nurse will be aware that giving potassium chloride to which patients are contraindicated: (choose all that apply)

A patient with chronic renal failure The patient taking a potassium sparing diuretic such as spironolactone

Which of the following patients should not receive heparin therapy due to increased risks? (select all that apply)

A patient with hemophilia A A patient with a BP of 195/100 A patient with a GI bleed

Match the Classification of drug with its use for specific conditions: PREVENT CVA, MI, DVT OR PE IN HIP SURGERIES

ANTICOAGULANT

A patient has been prescribed disulfiram. The nurse would most appropriately counsel the patient about which of the following?

A toxic reaction if alcohol is consumed

Match the Classification of drug with its use for specific conditions: PROSTHETIC HEART VALVES

ANTICOAGULANT

Match the drug with its classification: HEPARIN

ANTICOAGULANT

Match the following drug with its indication for use: ENOXAPARIN

ANTICOAGULANT

Match the medication with its classification: PSYLLIUM MUCOID

ABSORBS WATER AND INCREASES FECAL MASS

Match the following drug with its indication for use: VENLAFAXINE

ADJUVENT FOR PAIN

Match the following cancer and cancer treatment terms with the correct definition: CARCINOGENS

AGENTS THAT CAUSE CANCER

Match the following drug with its indication for use: WARFARIN

ANTICOAGULANT

Valporic acid can be used as an anticonvulsant and a vascular HA suppressant. This medcation

ALL OF THE ABOVE Rapidly distributes into plasma and extracellular water Crosses the blood-brain and maternal-fetal barriers Enters breast milk

Match the medications listed below with the complication of chemotherapy for which it is prescribed: EPOEITIN ALFA

ANEMIA

Match the Classification of drug with its use for specific conditions: AFTER PCI (PERCUTANEOUS CORONARY INTERVENTION)

ANTIPLATELET

Match the following drug with its indication for use: ABCIXMAB

ANTIPLATELET

Match the following drug with its indication for use: ASPIRIN (ACETYL SALYCILATE OR ASA)

ANTIPLATELET

Match the following drug with its indication for use: CLOPIDOGREL

ANTIPLATELET

Match the following Pharmacotherapy for coughs: DEXTROMETHORPHAN

ANTITUSSIVE: NONOPIOID

Match the following Pharmacotherapy for coughs: HYDROCODONE WITH HOMATROPINE

ANTITUSSIVE: OPIOID

Which lab value need to be assessed when a patient is taking acetaminophen

AST and ALT

Which lab value needs to be assessed when a patient is taking acetaminophen regularly?

AST and ALT

A client is starting on Zafirlukast. What is a priority nursing assessment before starting the drug?

AST and ALT lab values

Match the Classification of drug with its use for specific conditions:ATRIAL FIBRILLATION

ATNICOAGULANT

Irritable bowel syndrome is a common disorder of the lower GI tract which includes which of the following symptoms? Check all that apply

Abdominal pain and bloating. Mucus in the stool Altered bowel habits, with diarrhea alternating with constipation

One of your assigned patients is being started on clopidogrel (Plavix), an anti-platelet drug, following a TIA (transient ischemic attack). The reason he is receiving this drug is for its

Ability to alter platelet membranes

What is the most common adverse effect of Reteplase?

Abnormal bleeding

At what age is it recommended that the annual influenza immunization be started for a healthy child?

6 months

At what age should an adult et the herpes zoster vaccination (HZV)?

60 years, regardless of wether they had a prior episode of herpes zoster

At what age is it recommended that adults receive the pneumococcal vaccine?

65 years

Patients should not take zolpidem unless able to stay in bed for..........before being active again.

7-8 hours

What is contained in Humulin 70/30 or Novolin 70/30 premixed insulin combination?

70% NPH insulin and 30% regular insulin

Prevalence of prescription drug use among older adults increases substantially with age. What percentage of people 65 and older use at least 1 drug per week?

90%

Serum levels of unbound (free) drug ma increase in which of the following patients?

A 72 year old who has stopped eating due to grief related to loss of a loved one.

What is the action of psyllium mucilloid?

Absorbs water and increases fecal mass, thereby providing peristalsis in the bowel to have substance to push against

Terminology: Which of the following terms describe medications used "to relieve pain?"

Analgesics

Which of the following drugs inhibits warfarin metabolism?

Acetaminophen

Which over-the-counter medications are hepatotoxic and can cause liver tissue destruction when taken in large amounts? Select all that apply

Acetaminophen NSAIDS

An elderly nursing home resident have been diagnosed with type 2 diabetes, hypertension, and dementia. The patient begins taking an antipsychotic drug for agitation. Tremor and bradykinesia develop, so an anticholinergic is added to the drug regimen. Within 3 days the patient displays a marked cognitive deficit. Which medication is the most likely cause of the cognitive change?

Acetylcholine

What is true of calcitonin

Acts in opposition to Vit. D Acts in opposition to PTH Increase bone density Can be used to treat hypercalcemia in the blood

Identify risks for osteoporosis: Select all that applu

Advancing age Family history of osteoporosis High alcohol consumption Testosterone decline

When are type 2 anti diabetic drugs usually prescribed?

After diet, exercise and on anti diabetic drug have failed to reduce blood glucose to normal levels

Providing safe and effective drug therapy for the elderly. Why is it so difficult?

Aging can alter pharmacodynamics and pharmacokinetics

What is the most serious side effect of a penicillin drug?

Anaphylaxis

A patient on chemotherapy states he no longer wants to eat. Which of the following would be the best for this client?

Albumin 2.5 g/dL

Your client has 3 new inhalers, and you are teaching her about them. You will explain to her that the following inhaler is considered a rescue inhaler to be used for an acute asthma attack.

Albuterol

A friend of yours is asking you about inhalers to prevent "asthma" caused by exercise. You explain to your friend that

Albuterol inhalers are used 15 to 30 minutes prior to physical activity to prevent exercise-induced bronchospasm.

With which condition would dextromethorphan (Delsym) be most successful in suppressing a cough?

Allergic rhinitis

Instructions for administering Phenytoin IV include:

Always use a filter when infusing Phenytoin (traces of dextrose in an existing line can cause microscopic precipitate formation which become emboli if infused.

Match the Antibiotic with the class that it belongs to: NEOMYCIN

Aminoglycosides

Match the antibiotic with the class that it belongs to: GENTAMICIN

Aminoglycosides

Which class of antibiotics is not absorbed enterally?

Aminoglycosides

Escitalopram may cause serious, potentially fatal reactions when used with .....................(Select all that apply)

Amitriptyline Duloxetine St. John's Wort Sertraline

One of the most important pharmacokinetic changes associated with aging is related to renal elimination of drugs. After age 30, what happens in the kidneys"

Creatinine clearance decreases

Match the cholesterol lowering agent with patient teaching instructions for taking it: NIACIN

TAKE WITH COLD WATER TO DECREASE FLUSHING, AND CONSIDER A BABY ASPIRIN AS WELL

Anticholinergic medications, which include antihistamines, cause which of the following?

Dry mouth (Xerostomia) and delirium

Morphine and hydromorphone are used for pain relief, but they also may be used to relieve which of the following symptoms during end-of-life care?

Dyspnea

Tretinoin is a topical agent commonly used to treat acne. Nursing considerations with this drug include:

Explaining that medication should not be applied until at least 20-30 minutes after washing

Which of the following medications act by stimulating the release of insulin from pancreatic islet cells? Check all that apply.

Glipizide (Glucotrol) Nateglinide (Starlix) Glimepiride (Amaryl)

low molecular weight heparin

Has a longer duration of action and is more stable than heparin

Your client is taking aspirin, ketorolac, and a topical hydrocortisone. He reports ringing in his ears for the last day. What is the priority action you will take?

Hold the next does of aspirin and report the symptom to the health care provider

The patient receives warfarin (Coumadin). The nurse notes that the patient's morning international normalized ratio (INR) is 7. What is the priority nursing intervention at this time?

Hold the next does of warfarin (Coumadin) and contact the physician.

Your client is taking aspirin (ASA), ketorolac (Toradol), and a topical hydrocortisone> He reports ringing in his ears for the last day. What is the priority action you will take?

Hold the next dose of aspirin and report the symptom to the health care provider

Incretins are:

Hormones secreted by the mucosa of the small intestine following a meal

A patient complains of cramps in his calves, paresthesia to his toes and a feeling that his heart is not beating normally. These symptoms may indicate which of the following imbalance?

Hyperkalemia

The hospitalized patient is receiving spironolactone. A consulting physician sees the patient and orders lisinopril. what is the primary assessment by the nurse?

Hyperkalemia

When teaching a patient about Digoxin toxicity, the nurse determine learning has occurred when the patient makes which statement?

I can eat a banana ever morning.

A nurse is teaching a patient newly started on antidysrhythmic medication. the nurse knows teaching has been effective when the patient states:

I may experience more dysrhythmias because of changes in electrolyte absorption

The patient is on an anti-hypertensive medication. The nurse knows patient teaching has been successful when the patient states:

I should change position from laying to standing slowly

A nurse is teaching a patient about her new prescription enalapril for hypertension. The nurse knows teaching was successful when the patient states

I will let my doctor know if I get dizzy, light headed or have a headache

After completing medication teaching to a teenage girl about beclomethasone inhaler, she makes the following statements. Which statement indicates more teaching is needed.

I will use this inhaler when I am having an asthma attack

Match with action of medication: GLUCAGON

INCREASES BLOOD GLUCOSE LEVELS

Match the antibiotic with the class that it belongs to: FLUTICASONE

INTRANASAL CORTICOSTEROID

Match the medication with its classification: SENNA

IRRITATES THE BOWEL AND CAUSES INCREASED PERISTALSIS TO MOVE FECES ALONG THE GI TRACT

Match the following drug with its indication for use: CETIRIZINE

ITCHING

In what routes and formularies can potassium chloride replacements be give to a patient with potassium of 3.1 choose all that apply

IV drip, tablet, oral powder to mixed in liquid

Patients taking levothyroxine as a thyroid medication are to take this medication:

In the morning 1/2 to 1 hour before breakfast

Your patient is taking the antipsychotic medications clozapine and divalproex sodium (Depakote), both of which can contribute to weight gain. For what should this patient be carefully monitored?

Increased blood glucose

When taking Metformin, alcohol.......................

Increases the risk for lactic acidosis

Promethazine (Phenergan) carries some heavy warning including: Choose all that apply.

Inducing a coma Exacerbating benign prostatic hyperplasia Additive effects with other CNS depressants Use in patients with closed-angle glaucoma Inducing a hypertensive crisis (ALL OF THE ABOVE)

Mr. Jones, a client of yours is on a regimen of warfarin (Coumadin) because of a venous thrombosis (blood clot) which developed following his orthopedic surgery. Which of the following blood tests will he likely need as he is monitored during his treatment?

Prothrombin time/INR

Which drug class would be used as first-line therapy for gastroesophageal reflux disease (GERD)?

Proton pump inhibitors

A client goes to the store to get some over the counter cold medication at night when the pharmacy is close. HE is looking for the pseudophedrine, which is locked behind the pharmacy door. He must wait until the pharmacy opens in the morning. What is the reason for this?

Pseudoephedrine is the starting ingredient for methamphetamines

Which statement is not true concerning the reason crystalloids are given to patients"

Pull fluid into the vascular space to help pressure in the blood vessels (veins) increase

Match the following classes of drugs with how they are used to relieve symptoms and avoid complications of COPD:ANTIBIOTICS

REDUCE INFECTION

Match the following classes of drugs with how they are used to relieve symptoms and avoid complications of COPD: CORTICOSTEROIDS

REDUCE INFLAMMATION

Match the following classes of drugs with how they are used to relieve symptoms and avoid complications of COPD: EXPECTORANTS

REDUCE VISCOSITY OF BRONCHIAL MUCOUS

Match the following classes of drugs with how they are used to relieve symptoms and avoid complications of COPD: MUCOLYTICS

REDUCE VISCOSITY OF BRONCHIAL MUCOUS

Match the following drug with its indication for use: NALOXONE

RESPIRATORY RATE OF 8

A patient receiving allopurinol for gout symptoms asks why he should decrease his alcohol intake. The nurse's response is based on the knowledge that alcohol:

Raises uric acid levels.

What will the nurse teach the client to prevent one major side effect of a beclomethasone inhaler?

Rinse the mouth after each use of the inhaler and spit out after rinsing

Match the following medications with the most accurate drug classification: ASPIRIN

SALICYLATE

MATCH: Drugs are classified into five distinct categories or schedules, depending on the drug's acceptable medical use and the drug's abuse or dependency potential. Arrange the following drugs in ascending order, from most addictive to least addictive, according to their potential for abuse, according to the DEA (Drug Enforcement Agency): HEROINE

SCHEDULE I DRUG

MATCH: Drugs are classified into five distinct categories or schedules, depending on the drug's acceptable medical use and the drug's abuse or dependency potential. Arrange the following drugs in ascending order, from most addictive to least addictive, according to their potential for abuse, according to the DEA (Drug Enforcement Agency): HYDROMORPHONE

SCHEDULE II DRUG

MATCH: Drugs are classified into five distinct categories or schedules, depending on the drug's acceptable medical use and the drug's abuse or dependency potential. Arrange the following drugs in ascending order, from most addictive to least addictive, according to their potential for abuse, according to the DEA (Drug Enforcement Agency): CODEINE

SCHEDULE III DRUG

MATCH: Drugs are classified into five distinct categories or schedules, depending on the drug's acceptable medical use and the drug's abuse or dependency potential. Arrange the following drugs in ascending order, from most addictive to least addictive, according to their potential for abuse, according to the DEA (Drug Enforcement Agency): TRAMADOL

SCHEDULE IV DRUG

MATCH: Drugs are classified into five distinct categories or schedules, depending on the drug's acceptable medical use and the drug's abuse or dependency potential. Arrange the following drugs in ascending order, from most addictive to least addictive, according to their potential for abuse, according to the DEA (Drug Enforcement Agency): LOPERMIDE (LOMOTIL)

SCHEDULE V DRUG

Choose the actions of the following medications: DIPHENOXYLATE WITH ATROPINE (LOMOTIL)

SLOWS PERSTALSIS AND THEREBY TREATS DIARRHEA

Match the medication with its classification: SENNA

STIMULANT LAXATIVE

Mach the drug to the mechanism of action: GLIPIZIDE

STIMULATES RELEASE OF INSULIN FROM THE PANCREAS

Mach the drug to the mechanism of action: REPAGLINIDE

STIMULATES RELEASE OF INSULIN FROM THE PANCREAS

Match the Antibiotic with the class that it belongs to: SULFASOXAZOLE-TRAMETHOPRIM

SULFONAMIDES

Match the medication with its classification: DOCUSATE SODIUM

SURFACTANT STOOL SOFTENER

Which of the following mechanisms is used to describe when a person drops his blood glucose in the middle of the night, but wakes up in the morning with a high blood glucose levels?

Samogyi phenomenon

Morphine, hydromorphone, and methadone are controlled by the DEA, and belong to which of the following schedules

Schedule II

Which of the following class of lipid lowering agents puts a patient at risk for the rare but serious side effects called rhabdomyolysis?

Statins

You are teaching a young woman who is taking Glipizide (glucotrol) for Type II Diabetes. She is wondering how this medication works. What will you teach her?

Stimulating the pancreas to produce more insulin

A client on pioglitazone calls the office complaining of swelling in her legs, SOB, and feeling more tired since starting the drug. What is the best instruction by the nurse?

Stop taking the drug and come in for an appointment with the health care provider

The patient is receiving sodium bicarbonate intravenously for correction of acidosis secondary to diabetic coma. The nurse assesses cyanosis, slow respirations and irregular pulse. What is the nurse's priority action?

Stop the infusion and notify the physician of the patient's alkalosis

A patient who is on ranitidine (zantac) for peptic ulcer disease (PUD) smokes a half pack of cigarettes and drinks alcohol daily. What education will the nurse give the patient?

Stop using alcohol

Triptans are available for patient administration in which of the following route? Choose all that apply:

Sublingual subcutaneous Orally disintegrating table rectal oral intranasal (ALL OF THE OBOVE)

A patient diagnosed with inflammatory bowel disease (IBD) like ulcerative colitis, which drug helps maintain or prolong remission by inhibiting prostaglandins and leukotrienes?

Sulfasalazine

Multiple medications are used to treat Crohn's disease and ulcerative colitis. Which of the following drugs would be used as a first step of treatment?

Sulfasalazine (Azulfidine)

Match the antibiotic with the class that it belongs to: SULFAMETHAZOLE

Sulfonamides

A client calls saying she seems to need to use more and more of the albuterol each day. What is the nurse's most important concern?

This could be an early sign of a serious asthma attack

The nurse completes medication education for the client receiving antihistamines. The nurse evaluates that learning has occurred when the client makes which statement? check all that apply

This medication could make me very sleepy I need to increase fluids while taking this medication

Instructions for taking alendronate include: (check all that apply)

This should not be taken with antacids, iron or other vitamin supplements Take this at the same time of the morning. Stay upright and do not lay down for 30 minutes to an hour after taking this medicine

Valproate acid has FDA-approved indications for: (select all that apply)

acute mania simple and complex absence seizures prevention of migraine headaches

When a patient is taking a traditional antipsychotic medication, the nurse should assess carefully for extrapyramidal side effects (EPSEs) particularly:

akathisia

A patient is taking diphenhydramine during allergy season for his sneezing, stuffy nose, and itching eyes. What product should the nurse instruct him to avoid

alcohol

Patients taking nitrates like nitroglycerine for chest pain (angina) should avoid taking

alcohol

How does clopidogrel prolong bleeding time

alters the plasma membrane of platelets

Epinephrine is the primary drug for which type of shock?

anaphylactic

The classification for Phenobarbital includes: Select all that apply.

anticonvulsants Sedative/hypnotic

A child has been diagnosed with enuresis. TCA imipramine has been prescribed for the child. The nurse understands that this medication is in which category?

antispasmodic

Generally, migraines present with which of the following symptoms

appear as a throbbing or pulsating pain in the head

Almost all insulin today is from which source

human

Why are ACE inhibitors a good option for treatment of hypertension?

have little effect on electrolytes

Verapamil should be used very carefully or not at all with which CV condition?

heart failure

Prior to giving metoprolol, what parameter should the nurse assess, confirm or double-check and assure are in the patient chart?

heart rate

When administering Digoxin the nurse knows it is important to:

check quality and rhythm of apical pulse for 1 minute

When doing a medication reconciliation of medications gathered from Mr. Allen's home, the nurse notices the patient takes digoxin and Ginseng. It will become important to:

check the patient's heart rate for potential digoxin toxicity

Most drugs used to treat Alzheimer's disease effect:

cholinergic pathways, enzymes, and receptors

Albumin is considered a blood product. It is also a:

colloid

Which patient has the greatest risk for overdose with a benzodiazepine? A patient who:

combines the drug with alcohol

Your Patient G.H., has a history of Parkinson's disease. He is concerned about his home schedule of Carbidopa-Levodopa. Education to this patient should include: (mark all that apply)

his primary care provider will be notified to discuss continuing medication administration his medication will be continued on the same schedule as a home the medication will be administered during waking hours

A woman has been using an OTC antacid, Mylanta for relief of gastric upset with heartburn. She receives hemodialysis 3 times a week. The nurse who monitors the patient each week will be looking for symptoms of:

hypermagnesemia

What is the primary adverse effect of insulin therapy?

hypoglycemia

A patient is started on atenolol to decrease blood pressure. The nurse knows teaching was successful when the patient states this medication works by:

decreasing heart rate

A patient is placed on hydrochlorothiazide for HTN. The nurse knows teaching was successful when the patient states this medication works by:

decreasing the fluid level

A patient with rapid-cycling bipolar disorder is not responding to lithium therapy. At the next multidisciplinary team meeting, the nurse should point out that many rapid0cycling bipolar patients have been effectively treated using:

divalproex

A client who has been taking buspirone for two months returns to the clinic for a follow-up. The nurse determines that the medication is effective if there is an absent display of....

feelings of panic, fear, and uneasiness

A patient is receiving dextran 40. The nurse knows a concern with rapid administrations is?

fluid overload

Some indications for giving sodium bicarbonate IV are: (Choose all that apply)

for a pH 7.1 After an overdose of medicines for example aspirin, TCA's or phenobarbital

Patients taking some statins should be instructed to avoid

grapefruit juice

When choosing a medication to help treat heart failure the nurse knows lisinopril is contraindicated because the patient:

has hyperkalemia


Related study sets

Ch. 5 - Title and Title Transfer

View Set

Chapter 28: Disorders of Cardiac Conduction and Rhythm

View Set